You are on page 1of 125

IASbabas 60 DAY PLAN- UPSC PRELIMS 2017

ENVIRONMENT (Subject Wise Compilation)

Q.1) Which of the following processes contributes maximum to the power generation in
India?

a) Hydel power
b) Thermal power
c) Nuclear power
d) Solar and wind power

Q.1) Solution (b)

The all India installed power generation capacity by November 2015 is 2,82,023.78 MW
comprising 1,96,204.44 MW thermal 42,623.42 MW hydro, 5.780 MW nuclear and
37,415.53 MW R.E.S.

Q.2) The government of India had launched a very ambitious initiative of Ultra Mega
Power Projects. Which of the following statements are correct about UMPPs?
1. They are large hydel power projects to tap the full potential of unused streams.
2. Each UMPP will have the capacity of generating 10,000 MW of power.
3. Till now 10 UMPPs have already been sanctioned by the Government of India.
Select the code from below:
a) 1 and 2
b) 2 and 3
c) 1 and 3
d) None of the above

Q.2) Solution (d)

The Government of India had launched an initiative for the development of coal-based Ultra
Mega Power Projects (UMPPs), each with a capacity of 4,000 MW. The objective of the
initiative is to ensure cheaper tariffs utilizing economies of scale, catering to the need of a
number of states and to mitigate the risk relating to tie up of land fuel, water and other
statutory clearances, etc. The projects are being awarded to the successful developers on
the basis of
tariff based competitive bidding route employing super critical technology; to tie-up for
necessary inputs and clearances such as provision of site, fuel through captive mining
blocks, water and in-principle environment and forest clearances, project-specific shell
companies (SPVs) are set up as wholly owned subsidiaries of the Power Finance Corporation
Ltd. (PFC) - the nodal agency for these projects. These SPVs, along with the various
clearances, etc. are subsequently transferred to the successful developer. Four UMPPs
namely Sasan in Madhya Pradesh, Mundra in Gujarat, Krishnapatnam in Andhra Pradesh

www.iasbaba.com Page 1
IASbabas 60 DAY PLAN- UPSC PRELIMS 2017
ENVIRONMENT (Subject Wise Compilation)

and Tilaiya in Jharkhand have already been awarded to successful bidders and are at
different stages of development.

Q.3) Which of the following statements are correct about National Domestic Efficient
Lighting Program?
a) It urged people to use CFLs in place of tube lights and filament bulbs.
b) It urged people to use only 5 star efficiency rating appliances.
c) It promoted rural electrification.
d) None of the above

Q.3) Solution (d)

Domestic Efficient Lighting Programme (DELP) was launched in January 2015 urging the
people to use LED bulbs in place of incandescent bulbs, tube lights and CFL bulbs as they are
more efficient, long lasting and economical in their life cycle duration.

Q.4) The rate of biomass production in an ecosystem is called productivity. Which of the
following statements are correct about it?
1. Net primary productivity (NPP) is less than Gross primary productivity (GPP).
2. Net Primary Productivity (NPP) is the available biomass for the consumption of
heterotrophs.
Select the code from following:
a) 1 only
b) 2 only
c) Both 1 and 2
d) Neither 1 nor 2

Q.4) Solution (c)

Primary production is defined as the amount of biomass or organic matter produced per
unit area over a time period by plants during photosynthesis. It is expressed in terms of
weight (g 2) or energy (kcal m2). The rate of biomass production is called productivity.
It is expressed in terms of g2 yr 1 or (kcal m2) yr1 to compare the productivity of
different ecosystems. It can be divided into gross primary productivity (GPP) and net
primary productivity (NPP). Gross primary productivity of an ecosystem is the rate of
production of organic matter during photosynthesis. A considerable amount of GPP is
utilised by plants
in respiration. Gross primary productivity minus respiration losses (R), is the net primary
productivity (NPP).
GPP R = NPP

www.iasbaba.com Page 2
IASbabas 60 DAY PLAN- UPSC PRELIMS 2017
ENVIRONMENT (Subject Wise Compilation)

Net primary productivity is the available biomass for the consumption to heterotrophs
(herbiviores and decomposers). Secondary productivity is defined as the rate of formation
of new organic matter by consumers.

Q.5) Which of the following statements correctly explains the process of catabolism?

a) Seeping of water soluble organic material into lower soil horizons.


b) Breaking up for detritus material into smaller fragments.
c) Breaking up of larger organic molecules into smaller inorganic molecules.
d) Use of inorganic molecules by plants to form larger organic molecules.

Q.5) Solution (c)

The important steps in the process of decomposition are fragmentation, leaching,


catabolism, humification and mineralisation.
Detritivores (e.g., earthworm) break down detritus into smaller particles. This process is
called fragmentation. By the process of leaching, watersoluble inorganic nutrients go down
into the soil horizon and get precipitated as unavailable salts. Bacterial and fungal enzymes
degrade detritus into simpler inorganic substances. This process is called as catabolism.

Q.6) Which of the following factors play an important role in determining the rate of
decomposition of detritus material?
1. Chemical composition of Detritus
2. Temperature
3. Soil moisture
Select the code from below:
a) 1 only
b) 2 and 3
c) 1 and 2
d) All of the above

Q.6) Solution (d)

The rate of decomposition is controlled by chemical composition of detritus and climatic


factors. In a particular climatic condition, decomposition rate is slower if detritus is rich in
lignin and chitin, and quicker, if detritus is rich in nitrogen and water-soluble substances like
sugars. Temperature and soil moisture are the most important climatic factors that regulate
decomposition through their effects on the activities of soil microbes. Warm and moist
environment favour decomposition whereas low temperature and anaerobiosis inhibit
decomposition resulting in build up of organic materials.

www.iasbaba.com Page 3
IASbabas 60 DAY PLAN- UPSC PRELIMS 2017
ENVIRONMENT (Subject Wise Compilation)

Q.7) Deep sea Hydrothermal vents were recently discovered by the scientist. Which of
the following statements are correct about Hydrothermal ecosystems?
1. The organisms are not dependent on sunlight for food production.
2. Bacteria create energy using hydrogen sulphide through the process of
chemosynthesis.
3. Only micro organisms can survive in this environment.

Select the code from below:


a) 1 and 2
b) 2 and 3
c) 1 and 3
d) All of the above

Q.7) Solution (a)

Deep-sea hydrothermal vents form as a result of volcanic activity on the ocean floor. Water
seeps through cracks in the Earth's crust, dissolving metals and minerals as it becomes
super-heated from nearby magma.
This water - which can reach temperatures of 400C - eventually rises back through the
ocean floor, erupting as a geyser from a hydrothermal vent. The dissolved minerals and
metals precipitate on contact with the cold sea water, forming a chimney around the vent.
When scientists first discovered these vents in the 1970s, they were amazed to find thriving
communities of shrimp, crabs, giant tubeworms, clams, slugs, anemones, and fish. These
rare geological features turned out to be oases on the otherwise sparsely inhabited ocean
floor, with a biomass equivalent to that of a rainforest.

Instead of sunlight, vent life relies on hydrogen sulfide - more commonly known as rotten
egg gas and toxic to most land-based life.
In a process called chemosynthesis, specialized bacteria create energy from the hydrogen
sulfide present in the mineral-rich water pouring out of the vents. These bacteria form the
bottom level of the food chain in these ecosystems, upon which all other vent animals are
dependent.

Q.8) Based on the source of their food, organisms occupy a special place in the food chain.
This position is called:
a) Niche
b) Trophic level
c) Horizon
d) Food web

www.iasbaba.com Page 4
IASbabas 60 DAY PLAN- UPSC PRELIMS 2017
ENVIRONMENT (Subject Wise Compilation)

Q.8) Solution (b)

Organisms occupy a place in the natural surroundings or in a community according to their


feeding relationship with other organisms.
Based on the source of their nutrition or food, organisms occupy a specific place in the food
chain that is known as their trophic level. Producers belong to the first trophic level,
herbivores (primary consumer) to the second and carnivores (secondary consumer) to the
third.

Q.9) Consider the following statements:


1. Pyramid of number is always upright because the number of primary producers is
always larger than the number of organisms dependent upon them.
2. Pyramid of energy is always upright because at each tropic level some of the energy
is lost as heat.
Which of the above statements are incorrect?

a) 1 only
b) 2 only
c) Both 1 and 2
d) Neither 1 nor 2

Q.9) Solution (a)

In most ecosystems, all the pyramids, of number, of energy and biomass are upright, i.e.,
producers are more in number and biomass than the herbivores, and herbivores are more in
number and biomass than the carnivores. Also energy at a lower trophic level is always
more than at a higher level.

But there are exceptions to this trend. In forests, the number of tress is less than the
number of organisms dependent on each tree(insects, birds etc.)
Even in pyramid of biomass, there are exceptions like sea ecosystem. Very large organisms
are dependent on phytoplanktons.
Pyramid of energy is always upright, can never be inverted, because when energy flows
from a particular trophic level to the next trophic level, some energy is always lost as heat at
each step. Each bar in the energy pyramid indicates the amount of energy present at each
trophic level in a given time or annually per unit area.

Q.10) Which of the following statements are correct about the process of secondary
succession?

www.iasbaba.com Page 5
IASbabas 60 DAY PLAN- UPSC PRELIMS 2017
ENVIRONMENT (Subject Wise Compilation)

1. Secondary succession is much slower than the primary succession.


2. Secondary succession begins in areas where natural vegetation has been destroyed.

Select the code from below:


a) 1 only
b) 2 only
c) Both 1 and 2
d) Neither 1 nor 2

Q.10) Solution (b)

Ecological succession, the process by which the structure of a biological community evolves
over time. Two different types of successionprimary and secondaryhave been
distinguished. Primary succession occurs in essentially lifeless areasregions in which the
soil is incapable of sustaining life as a result of such factors as lava flows, newly formed sand
dunes, or rocks left from a retreating glacier. Secondary succession occurs in areas where a
community that previously existed has been removed; it is typified by smaller-scale
disturbances that do not eliminate all life and nutrients from the environment.

Secondary succession begins in areas where natural biotic communities have been
destroyed such as in abandoned farm lands, burned or cut forests, lands that have been
flooded. Since some soil or sediment is present, succession is faster than primary
succession.
Description of ecological succession usually focuses on changes in vegetation. However,
these vegetational changes in turn affect food and shelter for various types of animals. Thus,
as succession proceeds, the numbers and types of animals and decomposers also change.
At any time during primary or secondary succession, natural or human induced disturbances
(fire, deforestation, etc.), can convert a particular seral stage of succession to an earlier
stage. Also such disturbances create new conditions that encourage some species and
discourage or eliminate other species.

Q.11) Phosphorus is an important element used in metabolism and as a building up


material in organisms. Which of the statements are correct for phosphorus cycle?
1. Rocks are the major reservoir of phosphorus which contains phosphorus in the form
of phosphates.
2. Animals obtain phosphorus indirectly through plants.
3. There is no respiratory release of phosphorus in atmosphere.
Select the code from following:
a) 1 and 2
b) 2 and 3

www.iasbaba.com Page 6
IASbabas 60 DAY PLAN- UPSC PRELIMS 2017
ENVIRONMENT (Subject Wise Compilation)

c) 1 and 3
d) All of the above

Q.11) Solution (d)

Phosphorus is a major constituent of biological membranes, nucleic acids and cellular


energy transfer systems. Many animals also need large quantities of this element to make
shells, bones and teeth. The natural reservoir of phosphorus is rock, which contains
phosphorus in the form
of phosphates. When rocks are weathered, minute amounts of these phosphates dissolve in
soil solution and are absorbed by the roots of the plants. Herbivores and other animals
obtain this element from plants. The waste products and the dead organisms are
decomposed by phosphate-solubilising bacteria releasing phosphorus. Unlike carbon cycle,
there is no respiratory release of phosphorus into atmosphere.

Q.12) The benefits that people obtain from ecosystems are termed as ecosystem services.
Which of the following are the types of Ecosystem services?
1. Provisioning services
2. Regulating services
3. Habitat services
4. Cultural services

Select the code from below:


a) 1,2 and 3
b) 2,3 and 4
c) 1 and 2
d) All of the above

Q.12) Solution (d)

Ecosystem services are the direct and indirect contributions of ecosystems to human well-
being.

Ecosystem services can be categorized in four main types:

Provisioning services are the products obtained from ecosystems such as food, fresh water,
wood, fiber, genetic resources and medicines.

www.iasbaba.com Page 7
IASbabas 60 DAY PLAN- UPSC PRELIMS 2017
ENVIRONMENT (Subject Wise Compilation)

Regulating services are defined as the benefits obtained from the regulation of ecosystem
processes such as climate regulation, natural hazard regulation, water purification and
waste management, pollination or pest control.

Habitat services highlight the importance of ecosystems to provide habitat for migratory
species and to maintain the viability of gene-pools.

Cultural services include non-material benefits that people obtain from ecosystems such as
spiritual enrichment, intellectual development, recreation and aesthetic values.

Q.13) Lichens are one of the most important pioneer species. Which of the following
statements are correct about Lichens?
1. Lichens are not single organisms, but they are formed by symbiotic relationship
between algae and Fungi.
2. Lichens are found only in high alpine areas where moisture is scarce.
Select the code from following:
a) 1 only
b) 2 only
c) Both 1 and 2
d) Neither 1 nor 2

Q.13) Solution (a)

A lichen is not a single organism. Rather, it is a symbiosis between different organisms - a


fungus and an alga or cyanobacterium.

The non-fungal partner contains chlorophyll and is called the photobiont. The fungal
partner may be referred to as the mycobiont. While most lichen partnerships consist of one
mycobiont and one photobiont, that's not universal for there are lichens with more than
one photobiont partner. When looked at microscopically, the fungal partner is seen to be
composed of filamentous cells and each such filament is called a hypha. These hyphae grow
by extension and may branch but keep a constant diameter. Amongst the photobionts there
are those that are also filamentous in structure while others are composed of chains or
clusters of more-or-less globose cells.

Given that they contain chlorophyll, algae and cyanobacteria can manufacture
carbohydrates with the help of light via the process of photosynthesis. By contrast, fungi do
not make their own carbohydrates. Every fungus needs existing organic matter from which

www.iasbaba.com Page 8
IASbabas 60 DAY PLAN- UPSC PRELIMS 2017
ENVIRONMENT (Subject Wise Compilation)

to obtain carbon. In a lichen some of the carbohydrate produced by the photobiont is of


course used by the photobiont but some is 'harvested' by the mycobiont.

Lichens occur from sea level to high alpine elevations, in many environmental conditions,
and can grow on almost any surface. Lichens are abundant growing on bark, leaves, mosses,
on other lichens, and hanging from branches "living on thin air" (epiphytes) in rain forests
and in temperate woodland. They grow on rock, walls, gravestones, roofs, exposed soil
surfaces, and in the soil as part of a biological soil crust. Different kinds of lichens have
adapted to survive in some of the most extreme environments on Earth: arctic tundra, hot
dry deserts, rocky coasts, and toxic slag heaps. They can even live inside solid rock, growing
between the grains.

Q.14) Bhopal gas tragedy has been one of the most deadliest industrial hazards in the
history of India. Which of the following gas was released causing the disaster?
a) Potassium cyanide
b) Potassium isocyanate
c) Methyl isocyanate
d) Phosgene

Q.14) Solution (c)

The Bhopal disaster or Bhopal gas tragedy was an industrial accident. It happened at a
Union Carbide subsidiary pesticide plant in the city of Bhopal, India. On 3 December 1984,
the plant released 42 tonne of toxic methyl isocyanate (MIC) gas, exposing more than
500,000 people to toxic gases.

A mixture of poisonous gases flooded the city, causing great panic as people woke up with a
burning sensation in their lungs. Thousands died immediately from the effects of the gas.
Many were trampled in the panic that followed. The first official immediate death toll was
2,259. Another estimate is that 8,000 died within two weeks, that an additional 8,000 have
since died from gas-related diseases.

Q.15) If a large amount of organic waste is dumped in a lake, it BOD will


a) Decrease
b) Increase
c) Remains unchanged
d) Can increase or decrease

Q.15) Solution (b)

www.iasbaba.com Page 9
IASbabas 60 DAY PLAN- UPSC PRELIMS 2017
ENVIRONMENT (Subject Wise Compilation)

Biochemical oxygen demand (BOD, also called biological oxygen demand) is the amount of
dissolved oxygen needed (i.e., demanded) by aerobic biological organisms to break down
organic material present in a given water sample at certain temperature over a specific time
period. The BOD value is most commonly expressed in milligrams of oxygen consumed per
litre of sample during 5 days of incubation at 20 C and is often used as a surrogate of the
degree of organic pollution of water.
With increase in organic waste the requirement of oxygen by the microbes to decompose it
will also increase.

Q.16) Since 1970, Earth Day is celebrated every year on 22nd April. Which of the following
statements are correct about Earth Day?

1. It aims to encourage people to carry out activities that will benefit the Earth, such as
recycling more, using solar power or plant trees.
2. People voluntarily switch off all electrical appliances from 8:30 9:30 pm to reduce
carbon footprint.
Select the code from below:
a) 1 only
b) 2 only
c) Both 1 and 2
d) Neither 1 nor 2

Q.16) Solution (a)

Earth Day is celebrated every year on April 22. It was first celebrated in 1970, and since then
it is celebrated in more than 193 countries by over a billion people every year. Earth Day
celebrations are coordinated globally by the Earth Day Network. Earth Day 2017s Campaign
is Environmental & Climate Literacy. Environmental and climate literacy was chosen as it is
important for creating green voters and advancing environmental and climate laws and
policies across the globe as well as to accelerate green technologies and jobs.

It aims to encourage people to carry out activities that will benefit the Earth, such as
recycling more, using solar power or plant trees.

Note: Switching off appliances is done during earth hour. Earth day and Earth hour are
two different events. Dont get confused between the two.

www.iasbaba.com Page 10
IASbabas 60 DAY PLAN- UPSC PRELIMS 2017
ENVIRONMENT (Subject Wise Compilation)

Q.17) Which of the following countries became the first country to ban all kinds of metal
mining in the country?

a) Papua New Guinea


b) El Salvador
c) Peru
d) Cuba

Q.17) Solution (b)

Central Americas smallest nation El Salvador became first country in the world to ban metal
mining nationwide. In this regard, it has passed a law that bans all mining for gold and other
metals. The law imposes broad prohibition on the extraction of minerals to protect the
nations allegedly quite fragile environment, water reservoirs and reduce social tensions.

The new law bans all exploration, extraction and processing of metals both in open pits and
mines It bans the use of cyanide and mercury for mining. It does not apply to quarrying or
the mining of coal, salt and other non-metallic resources.

Q.18) Kanha Tiger reserve is the first ever reserve in India to officially introduce a mascot
to create a better connect with the visitors. The mascot has been named Bhoorsingh.
Which animal does Bhoorsingh represent?

a) Tiger
b) Stag
c) Elephant
d) Rhinoceros

Q.18) Solution (b)

Kanha in Madhya Pradesh became first tiger reserve in India to officially introduce a mascot
named Bhoorsingh the Barasingha, to create a better connect with its visitors, especially the
younger generation. The mascot Bhoorsingh will soon feature in more artwork, posters and
merchandise for the park management at Kanha.

www.iasbaba.com Page 11
IASbabas 60 DAY PLAN- UPSC PRELIMS 2017
ENVIRONMENT (Subject Wise Compilation)

Barasingha (Stag) is the state animal of Madhya Pradesh.

Q.19) Environmental Activists across the world focused on a grave but less discussed issue
of Photo Pollution (Light Pollution). Which of the following statements are correct about
Photo Pollution?

1. Photo Pollution refers to blocking of natural light of moon at night due to dust and
smoke in Urban Areas.
2. It disturbs migratory pattern of birds which use stars to navigate.

Select the code from below:

a) 1 only
b) 2 only
c) Both 1 and 2
d) Neither 1 nor 2

Q.19) Solution (b)

Light pollution is artificial brightening of the night sky caused by man-made lightening
sources, which has a disruptive effect on natural cycles and inhibits the observation of stars
and planets. It is also known as photo pollution or luminous pollution and basically is the
misdirected or obtrusive of natural light by excessive artificial light. More than 80% of
humanity lives under skies saturated with artificial light.

There are different components of Light pollution:

Glare: excessive brightness that causes visual discomfort.


Skyglow: brightening of the night sky over inhabited areas.
Clutter: bright, excessive and confusing groupings of light sources.

www.iasbaba.com Page 12
IASbabas 60 DAY PLAN- UPSC PRELIMS 2017
ENVIRONMENT (Subject Wise Compilation)

Light trespass: light falling where it is not intended or needed.

Effects:
Disturbs the reproductive cycles of some animals.
Disturbs migration of birds that navigate using the stars and to disorient night-flying
insects.
In humans, it disturbs circadian rhythms that regulate hormones and other bodily
functions.
Excessive blue light emitted from LEDs directly affects sleep pattern in Human by
suppressing the production of the hormone melatonin, which mediates the sleep-
wake cycle in humans.
Q.20) The theme for Worlds Wetland Day is Wetlands for Disaster Risk Reduction.
Mangroves are important wet lands, which of the following roles are played by
Mangroves?
1. Help in reducing coastal erosion.
2. Acts as a buffer zone between sea surges and land during storms.
3. Reduces the intensity of Tsunami waves.
4. Helps in reducing the salinity of coastal waters

Select the code from below:


a) 1,2 and 3
b) 2,3 and 4
c) 2 and 3
d) All of the above

Q.20) Solution (a)

Mangrove forests along tropical coastlines and in salt water environments are an important
category of wetlands. They are a critical component of marine ecosystems, as they serve as
nursery grounds for many aquatic species, including commercially important fish species.
Mangroves also serve as excellent buffer zones between coastal lands and open ocean as
they reduce the impacts of storms and keep coastal erosion under control. Mangrove
forests also had reduced the impact of the 2004 Southeast Asian tsunami. These rich
mangroves ecosystems are threatened mostly by conversion into agriculture and
aquaculture (shrimp farms), urban and resort development and rising sea levels.

Q.21) India has recently ratified the second commitment period of Kyoto Protocol. Which
of the following statements are correct about Kyoto Protocol?
1. It was adopted in 1997 to fight Global warming.

www.iasbaba.com Page 13
IASbabas 60 DAY PLAN- UPSC PRELIMS 2017
ENVIRONMENT (Subject Wise Compilation)

2. The protocol is based on principle of Equity and Common but differentiated


responsibilities and respective capabilities.
3. All the signatories of the protocol have mandatory binding targets.
Select the code from below:
a) 1 and 2
b) 2 and 3
c) 1 and 3
d) All of the above

Q.21) Solution (a)

The Kyoto Protocol was adopted in 1997 to fight global warming by reducing GHGs emission
and came into effect in 2005.
The 1st commitment period under the Kyoto Protocol was from 2008-2012.
The 2nd commitment period for the period 2013- 2020 was adopted in 2012 by the
Doha Amendment of the Kyoto Protocol.
The protocol is based on principle of Equity and Common but differentiated
responsibilities and respective capabilities (CBDR).
It places obligations on developed nations to undertake mitigation targets to reduce
emissions by 5.2% of 1990 levels during 2008-2012 period) and provide financial
resources and technology to developing nations.
Developing countries like India have no mandatory mitigation obligations or targets
under the Kyoto Protocol.

Q.22) Consider the following statements:


1. In Meeting of Parties (MoP) of Montreal Protocol at Kigali, HFCs have been included
in the list of chemicals under Montreal Protocol.
2. HFCs are the severe Ozone depleting substances.
3. India has promised to completely stop the use and production of HFCs by 2020.

Which of the above statements are incorrect?


a) 1 only
b) 2 and 3
c) 3 only
d) All of the above

Q.22) Solution (b)

www.iasbaba.com Page 14
IASbabas 60 DAY PLAN- UPSC PRELIMS 2017
ENVIRONMENT (Subject Wise Compilation)

The Montreal Protocol of the Vienna Convention for Protection of Ozone Layer which
entered force in January 1989 aimed to phase out the ozone depleting substances (ODS).
At Kigali, it was decide to include HFCs in the list of chemicals under the Montreal Protocol
in order to regulate their production, consumption and phasing them out with time.
It also added mechanism to provide financial assistance from the Multilateral Fund created
under the Montreal Protocol. Under it, funding for R&D and servicing sector in developing
countries also was included in the agreed solutions on finance.
Kigali amendments to the Montreal Protocol also for the first time will incentivise
improvement in energy efficiency in case of use of new refrigerant and technology.
Note: HFCs are not ODS but potent global warming substances and controlling them can
contribute substantially to limit global temperature and advance actions for addressing
climate change.
In the Kigali meet, India had successfully negotiated the baseline years within a range of
2024 to 2030 and freeze year in subsequent years for phasing down the use of HFCs.

It had two set of baselines years were agreed for developing countries. India along with nine
other countries will have baseline years of 2024, 2025, 2026. India will completely phase
down of HFCs in 4 steps from 2032 onwards with cumulative reduction of 10% in 2032, 20%
in 2037, 30% in 2042 and 85% in 2047

It also gives additional HCFC allowance of 65% that will be added to the Indian baseline
consumption and production.

Other developing countries including China (largest producer of HFCs in the world), South
Africa and Brazil opted for 2020-22 baseline.

Q.23) Pygmy hog is the smallest specie of pig found in India and is critically endangered.
Which of the following statements are correct about Pygmy hog?

1. It is found in the grasslands of Rajasthan.


2. Pygmy hog sucking louse is a parasite found on Pygmy hog which is also critically
endangered.
3. Pygmy Hog conservation program was started in 1995 to prevent the extinction of
species.

Select the code from below:

a) 1 and 2
b) 2 and 3
c) 1 and 3
d) All of the above

www.iasbaba.com Page 15
IASbabas 60 DAY PLAN- UPSC PRELIMS 2017
ENVIRONMENT (Subject Wise Compilation)

Q.23) Solution (b)

The pygmy hog (Porcula salvania) is a critically endangered suid, previously spread across
India, Nepal, and Bhutan, but now only found in Assam. The current world population is
about 150 individuals or fewer.

They are about 55 to 71 cm (21.5 to 28 in) long and stand at 2030 cm (812 in), with a tail
of 2.5 cm (0.98 in). They weigh 6.611.8 kg (1526 lb). Their skin is dark brownish-black and
the hair is dark. Piglets are born grayish-pink, becoming brown with yellow stripes along the
body length. Their heads are sharply tapered and they have a slight crest of hair on their
foreheads and on the back of their necks. Adult males have the upper canines visible on the
sides of their mouths.

The pygmy hog is designated as a Schedule I species in India under the Wildlife Protection
Act, 1972 and offences against them invite heavy penalties.

The Pygmy Hog Conservation Programme (PHCP) was established in 1995. The PHCP was
established under the umbrella of a formal 'International Conservation Management and
Research Agreement' by the Durrell Wildlife Conservation Trust, the IUCN's Pigs, Peccaries
and Hippo Specialist Group, the Forest Department, Government of Assam, and the Ministry
of Environment and Forests, Government of India.

Haematopinus oliveri, known commonly as the pygmy hog-sucking louse, is a critically


endangered species of insect in the suborder Anoplura, the sucking lice. It is an ectoparasite
found only on another critically endangered species, the Pygmy hog.

Q.24) Recently World Sustainable Development Summit (WSDS) was inaugurated by


President Pranab Mukherjee. Consider the following statements regarding WSDS:
1. It was organized by The Energy and Resources Institute (TERI).
2. It underscored need for businesses and private sector to take lead in poverty
reduction and to ensure rapid and sustained adoption of Sustainable Development
Goals (SDGs).
Which of the above statements are correct?
a) 1 only
b) 2 only
c) Both 1 and 2
d) Neither 1 nor 2

Q.24) Solution (c)

www.iasbaba.com Page 16
IASbabas 60 DAY PLAN- UPSC PRELIMS 2017
ENVIRONMENT (Subject Wise Compilation)

President Pranab Mukherjee inaugurated the first edition of World Sustainable


Development Summit (WSDS) at Vigyan Bhawan in New Delhi. The summit was organised by
The Energy and Resources Institute (TERI) with the theme Beyond 2015: People, Planet and
Progress.

WSDS has replaced TERIs earlier called Delhi Sustainable Development Summit (DSDS). The
first DSDS was organised in 2005.

It underscored need for businesses and private sector to take lead in poverty reduction and
to ensure rapid and sustained adoption of Sustainable Development Goals (SDGs).

It had brought together Nobel laureates, decision-makers political leaders from around the
world to deliberate on issues related to sustainable development.

The aim of the summit is to provide various stakeholders a single platform in order to
provide long-term solutions for the benefit of the global community.

2016 WSDS also hosted events like World CEO Sustainability Summit, Thematic Tracks,
Media Colloquium, Youth Leaders and Greenovation Exhibition.

Q.25) A new species of Pika has been discovered in Sikkim. Which of the following
statements are correct about Pikas?

1. Pikas are the members of rat family.


2. Since they occupy colder regions they hibernate during winters.
3. It has the longest tail with respect to body in the animal kingdom.

Select the code from below:

a) 1 and 2
b) 2 and 3
c) 1 and 3
d) None of the above

Q.25) Solution (d)

Pikas are members of the rabbit family and look like tailless rats.
They live in the mountains or in temperate regions.
They are a keystone species and ecosystem engineers.
They do not hibernate unlike other mammalian species inhabiting such cold climates.
The common name pika is basically used for any member of the Ochotonidae family.

www.iasbaba.com Page 17
IASbabas 60 DAY PLAN- UPSC PRELIMS 2017
ENVIRONMENT (Subject Wise Compilation)

Q.26) According to the Botanical Survey of India (BSI), which of the following states have
the highest number of flowering plants?
a) Uttarakhand
b) Jammu and Kashmir
c) Kerala
d) Tamil Nadu

Q.26) Solution (d)


According to the recent publication released by the Botanical Survey of India (BSI), Tamil
Nadu accounts for the highest number flowering plants in the country.
The publication titled Endemic Vascular Plants of India also revealed that almost one of
every four species of flowering plants found in India are endemic to the country.

Of the total 18,259 flowering plants reported in the country, 4,303 (over 23%) are found
only in India or they are endemic to the country. Of these, Tamil Nadu accounts for the
highest number of species with 410, followed by Kerala (357 species) and Maharashtra (278
species).

Q.27) Udaipur Declaration under BRICS is targeted to increase the cooperation in the field
of:
a) Disaster Management
b) Soil Conservation
c) Trade of Living organism
d) Genetic Engineering

Q.27) Solution (a)

Udaipur Declaration
BRICS nations to set up a dedicated Joint Task Force for Disaster Risk Management for
regular dialogue, exchange, mutual support and collaboration among them.
The meeting was a new milestone in collaboration and cooperation among BRICS countries
in the field of disaster management.
The roadmap for implementation of the three-year Joint Action Plan for BRICS emergency
services (2016-18) was also finalised.

It focused on two themes:

(i) Flood Risk Management and

www.iasbaba.com Page 18
IASbabas 60 DAY PLAN- UPSC PRELIMS 2017
ENVIRONMENT (Subject Wise Compilation)

(ii) Forecasting of Extreme Weather Events in the context of Changing Climate.

Q.28) Wildlife Crime Control Bureau was established in 2007 to fight the organized
Wildlife crime in India. Which of the following statements regarding WCCB are correct?

1. It is a statutory body established by Amending the Wildlife (Protection) Act 1972.


2. It comes under the Ministry of Environment and Forest.
3. A Centralised Wildlife Crime data bank has been established by WCCB.

Select the code from below:

a) 1 and 2
b) 2 and 3
c) 1 and 3
d) All of the above

Q.28) Solution (d)

he Government of India constituted a statutory body, the Wildlife Crime Control Bureau
(WCCB) on 6 June 2007, by amending the Wildlife (Protection) Act, 1972, a special Act to
protect the wildlife in the country. WCCB would complement the efforts of the state
governments, primary enforcers of the Wildlife (Protection) Act, 1972 and other
enforcement agencies of the country.

Wildlife Crime Control Bureau is a statutory multi-disciplinary body established by the


Government of India under the Ministry of Environment and Forests, to combat organized
wildlife crime in the country. The Bureau has its headquarter in New Delhi and five regional
offices at Delhi, Kolkata, Mumbai, Chennai and Jabalpur; three sub-regional offices at
Guwahati, Amritsar and Cochin; and five border units at Ramanathapuram, Gorakhpur,
Motihari, Nathula and Moreh. Under Section 38 (Z) of the Wild Life (Protection) Act, 1972, it
is mandated to collect and collate intelligence related to organized wildlife crime activities
and to disseminate the same to State and other enforcement agencies for immediate action
so as to apprehend the criminals; to establish a centralized wildlife crime data bank; co-
ordinate actions by various agencies in connection with the enforcement of the provisions
of the Act; assist foreign authorities and international organization concerned to facilitate
co-ordination and universal action for wildlife crime control; capacity building of the wildlife
crime enforcement agencies for scientific and professional investigation into wildlife crimes
and assist State Governments to ensure success in prosecutions related to wildlife crimes;
and advise the Government of India on issues relating to wildlife crimes having national and
international ramifications, relevant policy and laws. It also assists and advises the Customs
authorities in inspection of the consignments of flora & fauna as per the provisions of Wild
Life Protection Act, CITES and EXIM Policy governing such an item.

www.iasbaba.com Page 19
IASbabas 60 DAY PLAN- UPSC PRELIMS 2017
ENVIRONMENT (Subject Wise Compilation)

Q.29) Consider the following statements about West Nile fever

1. It is a mosquito-borne infection
2. West Nile Virus (WNV) was first identified in Uganda

Select the correct statements

a) Only 1
b) Only 2
c) Both 1 and 2
d) Neither 1 nor 2

Q.29) Solution (c)

West Nile fever is a mosquito-borne infection by the West Nile virus. Approximately 80% of
West Nile virus infections in humans have few or no symptoms. In the cases where
symptoms do occurtermed West Nile fever in cases without neurological diseasethe
time from infection to the appearance of symptoms is typically between 2 and 15 days.
Symptoms may include fever, headaches, feeling tired, muscle pain or aches, nausea, loss of
appetite, vomiting, and rash.

WNV is found in temperate and tropical regions of the world. It was first identified in the
West Nile subregion in Uganda in 1937. Prior to the mid-1990s, WNV disease occurred only
sporadically and was considered a minor risk for humans, until an outbreak in Algeria in
1994, with cases of WNV-caused encephalitis, and the first large outbreak in Romania in
1996, with a high number of cases with neuroinvasive disease. WNV has now spread
globally, with the first case in the Western Hemisphere being identified in New York City in
1999; over the next five years, the virus spread across the continental United States, north
into Canada, and southward into the Caribbean islands and Latin America. WNV also spread
to Europe, beyond the Mediterranean Basin, and a new strain of the virus was identified in
Italy in 2012. WNV spreads on an ongoing basis in Africa, Asia, Australia, the Middle East,
Europe, Canada and in the United States. In 2012 the US experienced one of its worst
epidemics in which 286 people died, with the state of Texas being hard hit by this virus.

Source: http://indianexpress.com/article/technology/science/scientists-discover-a-way-
to-predict-the-time-and-intensity-of-the-west-nile-virus-4544592/

www.iasbaba.com Page 20
IASbabas 60 DAY PLAN- UPSC PRELIMS 2017
ENVIRONMENT (Subject Wise Compilation)

Q.30) Consider the following statements

1. Blue carbon is the carbon captured by the world's oceans and coastal ecosystems.
2. Blue Carbon Initiative is coordinated by Conservation International (CI), the
International Union for Conservation of Nature (IUCN) only
3. Coastal blue carbon ecosystems are found along the coasts of every continent except
Antarctica

Select the correct statements

a) 1 and 2
b) 2 and 3
c) 1 and 3
d) All of the above

Q.30) Solution (c)

Blue carbon is the carbon captured by the world's oceans and coastal ecosystems. The
carbon captured by living organisms in oceans is stored in the form of biomass and
sediments from mangroves, salt marshes, seagrasses and potentially algae.

Coastal blue carbon ecosystems are found along the coasts of every continent except
Antarctica. Mangroves grow in the intertidal zone of tropical and subtropical shores.
Countries with the highest areas of mangroves include Indonesia, Australia, Mexico, Brazil,
Nigeria, Malaysia, Myanmar, Papua New Guinea, Cuba, India, Bangladesh, and Mozambique.

Tidal marshes are intertidal ecosystems occurring on sheltered coastlines ranging from the
sub-arctic to the tropics, though most extensively in temperate zones, mainly in Europe,
North-America, Australia and in the higher latitudes of South-America and Africa.

Seagrass meadows are communities of underwater-flowering plants found in coastal waters


of all continents except Antarctica. More than 60 seagrass species are known to exist, and as
many as 10 to 13 of them may co-occur in tropical sites.

While several countries are embarking on efforts to better map and quantify these systems,
regional, and global maps of coastal blue carbon hot spots currently do not exist.

www.iasbaba.com Page 21
IASbabas 60 DAY PLAN- UPSC PRELIMS 2017
ENVIRONMENT (Subject Wise Compilation)

Q.31) Ministry of Environment and Forest notified a new E Waste policy for 2016. Which
of the following statements are correct about this new E waste Policy?

1. Manufacturer, dealer, refurbisher and Producer Responsibility Organization (PRO)


have been intoruced as additional stakeholders in the rules.
2. Compact Fluorescent Lamp (CFL) and other mercury containing lamp brought under
the purview of rules.
3. Collection mechanism based approach has been adopted to include collection
centre, collection point, take back system etc for collection of e - waste by Producers
under Extended Producer Responsibility (EPR).
Select the code from below:

a) 1 and 2
b) 2 and 3
c) 1 and 3
d) All of the above

Q.31) Solution (d)

E - Waste (Management) Rules, 2016 - Whats New?

Manufacturer, dealer, refurbisher and Producer Responsibility Organization (PRO)


have been intoruced as additional stakeholders in the rules.

www.iasbaba.com Page 22
IASbabas 60 DAY PLAN- UPSC PRELIMS 2017
ENVIRONMENT (Subject Wise Compilation)

The applicability of the rules has been extended to components, consumables,


spares and parts of EEE in addition to equipment as listed in Schedule I.
Compact Fluorescent Lamp (CFL) and other mercury containing lamp brought under
the purview of rules.
Collection mechanism based approach has been adopted to include collection
centre, collection point, take back system etc for collection of e - waste by Producers
under Extended Producer Responsibility (EPR).
Option has been given for setting up of PRO, e - waste exchange, e - retailer, Deposit
Refund Scheme as additional channel for implementation of EPR by Producers to
ensure efficient channelization of e - waste.
Provision for Pan India EPR Authroization by CPCB has been introduced replacing the
state wise EPR authorization.
Collection and channelisation of e - waste in Extended Producer Responsibility -
Authorisation shall be in line with the targets prescribed in Schedule III of the Rules.
The phase wise Collection
Target for e - waste, which can be either in number or Weight shall be 30% of the
quantity of waste generation as indicated in EPR Plan during first two year of
implementation of rules followed by 40% during third and fourth years, 50% during
fifth and sixth years and 70% during seventh year onwards.
Deposit Refund Scheme has been introduced as an additional economic instrument
wherein the producer charges an additional amount as a deposit at the time of sale
of the electrical and electronic equipment and returns it to the consumer along with
interest when the end of - life electrical and electronic equipment is returned.
The e - waste exchange as an option has been provided in the rules as an
independent market instrument offering assistance or independent electronic
systems offering services for sale and purchase of e waste generated from end of
- life electrical and electronic equipment between agencies or organizations
authorised under these rules.
The manufacturer is also now responsible to collect e - waste generated during the
manufacture of any electrical and electronic equipment and channelise it for
recycling or disposal and seek authorization from SPCB.
The dealer, if has been given the responsibility of collection on behalf of the
producer, need to collect the e - waste by providing the consumer a box and
channelize it to Producer.
Dealer or retailer or e - retailer shall refund the amount as per take back system or
Deposit Refund Scheme of the producer to the depositor of e - waste.
Refurbisher need collect e - waste generated during the process of refurbishing and
channelise the waste to authorised dismantler or recycler through its collection
centre and seek one time authorization from SPCB.

www.iasbaba.com Page 23
IASbabas 60 DAY PLAN- UPSC PRELIMS 2017
ENVIRONMENT (Subject Wise Compilation)

The roles of the State Government has been also introduced in the Rules in order to
ensure safety, health and skill development of the workers involved in the
dismantling and recycling operations.

Q.32) Ploughing is an important agricultural activity to upturn the soil and prepare it for
seeding. Which of the following is not a result of ploughing?

a) It increases the rate of biological decay of organic material in the soil.


b) It makes the soil vulnerable for soil erosion.
c) It increases the aeration of the soil.
d) It causes Eutrophication.

Q.32) Solution (d)

Ploughing increases soil aeration and porosity and decreases soil compaction and kills
weeds. So this would increase crop growth and yield. However, ploughing, especially if done
improperly, can also create a hardpan below the soil surface, increase weeds (by bringing up
the weed seeds closer to the surface for easier germination) and destroy soil structure. So
ploughing can also reduce crop growth and yield.

When the soil is ploughed, this increases the rate of organic matter decomposition which
releases CO2, a greenhouse gas, into the atmosphere at a faster rate. It also reduces the
organic content in the soil.

Loose soil is also prone to soil erosion.

Note: Eutrophication is the excess growth of vegetation in a water body (Pond, Lake etc)
due to addition of nutrients in the water from run - off. Eutrophication is not directly related
to ploughing.

Q.33) Which of the following statements correctly explains Nuclear Winter?

a) It refers to the cool down period of nuclear reactor before new nuclear fuel is fed to
the reactor.
b) It refers to a condition of post nuclear war when sky is covered with smoke and dust
and does not allow sunlight to reach the surface. This causes a dip in temperature.
c) It refers to the areas of low temperature on stars (dark spot on Sun) when nuclear
fusion gets slowed down.
d) It refers to gradual decrease of the time period of winter season in tropical areas.

www.iasbaba.com Page 24
IASbabas 60 DAY PLAN- UPSC PRELIMS 2017
ENVIRONMENT (Subject Wise Compilation)

Q.33) Solution (b)

The nuclear winter scenario assumes that 100 or more city firestorms are ignited by the
nuclear explosions of a nuclear war, and the firestorms lift large enough amounts of sooty
smoke into the upper troposphere and lower stratosphere. Soot lifted by the movement
offered by the pyrocumulonimbus clouds that form during the blasts will cover the sky for
months and will not allow the sunlight to reach the surface causing a sudden and prolonged
drop in temperature.

Q.34) Which of the following statements are correct about the process of Global
warming?

1. It is a natural process.
2. The process of global warming is succeeded by the process of global cooling.
3. Anthropogenic activities have increased the rate of global warming.

Select the code from below:

a) 1 and 2
b) 2 and 3
c) 1 and 3
d) All of the above

Q.34) Solution (d)

Global warming is the term used to describe a gradual increase in the average temperature
of the Earth's atmosphere and its oceans, a change that is believed to be permanently
changing the Earths climate. There is great debate among many people, and sometimes in
the news, on whether global warming is real (some call it a hoax). But climate scientists
looking at the data and facts agree the planet is warming. While many view the effects of
global warming to be more substantial and more rapidly occurring than others do, the
scientific consensus on climatic changes related to global warming is that the average
temperature of the Earth has risen between 0.4 and 0.8 C over the past 100 years. The
increased volumes of carbon dioxide and other greenhouse gases released by the burning of
fossil fuels, land clearing, agriculture, and other human activities, are believed to be the
primary sources of the global warming that has occurred over the past 50 years. Scientists
from the Intergovernmental Panel on Climate carrying out global warming research have
recently predicted that average global temperatures could increase between 1.4 and 5.8 C

www.iasbaba.com Page 25
IASbabas 60 DAY PLAN- UPSC PRELIMS 2017
ENVIRONMENT (Subject Wise Compilation)

by the year 2100. Changes resulting from global warming may include rising sea levels due
to the melting of the polar ice caps, as well as an increase in occurrence and severity of
storms and other severe weather events.

Naturally earth moves from a cycle of global warming to Global Cooling. The cycle of global
warming ends up melting the polar ice and glaciers increasing the sea level. Since ocean is
the biggest carbon sink, it starts absorbing the greenhouse gases at an increased rate and
slowly the temperature starts falling and a cooling cycle starts. This results in an ice age.

Q.35) Which of the following statements are correct about the invasive species?

1. An invasive species is a non-native organism that causes ecological harm after being
introduced to a new environment.
2. Invasive species can cause extinction of endogenic species.
3. Invasive species are always a predator which reduces population of local species.

Select the code from below:

a) 1 and 2
b) 2 and 3
c) 1 and 3
d) All of the above

Q.35) Solution (a)

An invasive species is a non-native organism that causes ecological harm after being
introduced to a new environment.
Once they enter a new ecosystem, they can outcompete native organisms for resources like
food, especially if they lack natural predators.
Some invasive species also carry diseases that kill native organisms and many will consume
native plants and animals. Invasive species can ultimately cause the decline or extinction of
native species, decreasing biodiversity in an ecosystem.
Humans are responsible for the spread of a majority of earths invasive species, often
carrying them to different parts of the world on ships.
Invasive species are not necessarily predating.

Q.36) You must have observed the white trails in sky behind the jets. What exactly are
these white trails?

a) Dense smoke formed due to partial burning of fuel.


b) It is a trail of condensed water and ice crystals.
c) Jet streams which the plane is following

www.iasbaba.com Page 26
IASbabas 60 DAY PLAN- UPSC PRELIMS 2017
ENVIRONMENT (Subject Wise Compilation)

d) None of the above

Q.36) Solution (b)

Those white streaks planes leave behind are actually artificial clouds. They're called
contrails, which is a shortened version of the phrase condensation trail."

Airplane engines produce exhaust, just like car engines do. As hot exhaust gases escape
from a plane, the water vapor in the fumes hits the air. At heights of 26,000 feet or more,
the air is extremely cold (sometimes more than -40 F).

The cold air causes the water vapor to condense. This means the water vapor gases turn
into tiny water droplets or even freeze into tiny ice crystals before eventually evaporating.
This condensed water vapor and mixture of ice crystals make up the cloud-like trails you see
in the sky.

Q.37) Stratospheric Ozone protects us from harmful ultraviolet radiations while the
tropospheric Ozone is hazardous to living beings. Which of the following statements are
correct about tropospheric ozone?

1. It is a secondary pollutant as it is not released directly in the atmosphere by any


source.
2. Excess ozone causes the plants to close their stomata and reduces the rate of
Photosynthesis.
3. It is formed by the reaction of Volatile Organic Compounds and SOx during
photochemical smog.

Select the code from below:

a) 1 and 2
b) 2 and 3
c) 1 and 3
d) All of the above

Q.37) Solution (a)

Ozone (O3) is a constituent of the troposphere (it is also an important constituent of some
regions of the stratosphere commonly known as the ozone layer). The troposphere extends
from the Earth's surface to between 12 and 20 kilometers above sea level and consists of
many layers. Ozone is more concentrated above the mixing layer, or ground layer. Ground-

www.iasbaba.com Page 27
IASbabas 60 DAY PLAN- UPSC PRELIMS 2017
ENVIRONMENT (Subject Wise Compilation)

level ozone, though less concentrated than ozone aloft, is more of a problem because of its
health effects.

The majority of tropospheric ozone formation occurs when nitrogen oxides (NOx), carbon
monoxide (CO) and volatile organic compounds (VOCs), react in the atmosphere in the
presence of sunlight. NOx, CO, and VOCs are called ozone precursors. Motor vehicle
exhaust, industrial emissions, and chemical solvents are the major anthropogenic sources of
these chemicals.

Note: Sulphur Oxides are not involved in photochemical smog.

Q.38) The development of the National Waterway 1 is considered to be a severe threat for
the survival of Gangetic dolphins. Which of the following statements are correct about the
river dolphins of India?

1. In India the river dolphins are found only in Ganga river.


2. The dolphins have become blind because of the pollution of the river.
3. They rely on bio sonar to navigate and catch prey.

Select the code from below:

a) 1 and 2
b) 2 and 3
c) 3 only
d) All of the above

Q.38) Solution (c)

The development NW1 in the River Ganga is considered as single-largest threat to the
survival of the Gangetic Dolphins, whose numbers are declining in most parts of their
natural habitat.
River dolphins are found in Ganga and Brahmaputra and its tributaries. It has been declared
endangered species and is protected under Schedule I of the Wildlife Protection Act, 1972.

This fresh water dolphin species are practically blind. They rely on bio-sonar method to
move around and catch their prey.

Note: They are naturally blind. This has got nothing to do with the pollution of the river.

www.iasbaba.com Page 28
IASbabas 60 DAY PLAN- UPSC PRELIMS 2017
ENVIRONMENT (Subject Wise Compilation)

Q.39) The National Mission on Bioeconomy has been launched by the Institute of Bio
resources and Sustainable Development to boost rural economy by utilizing renewable bio
resources. In which of the following states has it been launched?
a) Uttarakhand
b) Assam
c) Meghalaya
d) Chattisgarh

Q.39) Solution (c)

The National Mission on Bioeconomy has launched in Shillong, Meghalaya by the Institute of
Bio-resources and Sustainable Development (IBSD). The mission is unique to south-east Asia
and India has become one of the few countries to tap bio-resources.

The purpose of the mission is to boost rural economy by utilizing bio-resources. Besides, it
can be optimally utilized can create a large number of jobs at village level. It also focuses on
sustainable utilization of renewable biological resources for food, bio-energy and bio-based
products through knowledge-based approaches. It has potential to generate new solutions
for the planets major challenges in the field of energy, food, health, water, climate change
and deliver social, economic and environmental benefits.

Q.40) The Union Government has launched National Green Highway Mission (NGHM).
Which of the following statements are correct about NGHM?
1. Aims to provide a green canopy along 100,000 km of highways.
2. It also aims to provide jobs for 1 million youth.
3. It also aims to plant marijuana (Weed) on both sides of the highway for therapeutic
use.
Select the code from the following:
a) 1 and 2
b) 2 and 3
c) 1 and 3
d) All of the above
Q.40) Solution (a)

NGHM was launched under the Green Highways Policy, 2015 to provide a holistic vision of
developing eco-friendly and green National Highways.
The mission aims to provide a green canopy along 100,000 km of highways and create jobs
for 1 million youth.

www.iasbaba.com Page 29
IASbabas 60 DAY PLAN- UPSC PRELIMS 2017
ENVIRONMENT (Subject Wise Compilation)

It has been initiated to engage corporates, Public Sector units, Government organizations
and other institutions for developing green corridor along National Highways through
plantation and allied activity on median, avenue and other available nearby land patches .
The funds for the mission will be met by Green Fund corpus which is 1 per cent of the total
project cost set aside for plantation purposes

Potential: The afforestation across National Highways is expected to help in sequestering


approximately 12 lakh metric ton carbon annually.
It will also generate jobs and can be a game-changer for agriculture and rural economy. In
future, the mission will be linked with the Mahatma Gandhi National Rural Employment
Guarantee scheme.

Q.41) Prime Minister recently released a book The birds of Banni Grassland. The Birds of
Banni Grassland is book based on environment and consists of a compilation of the
research work, relating to more than 250 species of birds found in the Banni area. In
which of the following states does this area come?

a) Rajasthan
b) Gujarat
c) Madhya Pradesh
d) Karnataka

Q.41) Solution (b)

Prime Minister Narendra Modi released a book titled The Birds of Banni Grassland in New
Delhi. The book was presented to the Prime Minister by the scientists of Gujarat Institute of
Desert Ecology (GUIDE). The Birds of Banni Grassland is book based on environment and
consists of a compilation of the research work, relating to more than 250 species of birds
found in the Banni area of Kutch, Gujarat.

Q.42) In order to reduce exhaust by fossil fuels, which of the following countries have
developed Worlds first electric road?
a) Finland
b) Norway
c) Sweden
d) Denmark

Q.42) Solution (c)

www.iasbaba.com Page 30
IASbabas 60 DAY PLAN- UPSC PRELIMS 2017
ENVIRONMENT (Subject Wise Compilation)

Sweden inaugurated a test stretch of an electric road in Sandviken to conduct tests with
electric power for hybrid heavy transports on public roads.

The project is being partially funded by Swedens three government agencies viz. Swedish
Transport Administration, Swedish Energy Agency, and the countrys innovation agency
Vinnova. The rest funding is provided by the private participants.

Q.43) Haryana Government has launched Asias first Gyps Vulture Reintroduction
Programme at Jatayu conservation breeding center in Pinjore. Which of the following
drugs are responsible for the large scale decline in Vulture population?

a) DDT
b) Diclofenac
c) Oxytocin
d) Urea

Q.43) Solution (b)

The main reason for rapid decline in population of vulture is due to Diclofenac, a non-
steroidal anti-inflammatory drug which is given to cattle in inflammation and pain.
Though vultures have robust digestive system, they are not able to break down Diclofenac
and die of renal failure or kidney failure after eating carcasses of cattle administered with
the drug.

Q.44) The Floating gardens of the Dal lake in Kashmir are famous for their vegetable
production. The name given to these floating gardens is?

a) Phumdi
b) Raad
c) Bet
d) Tila

Q.44) Solution (b)

Phumdi: The floating islands of Loktak lake in Manipur are called Phumdis

Raad: Raad is the local name of floating gardens in Kashmir.

Bet: Bet is the name given to a flood plain when it is flooded with water. Sometimes the
entire flood plain is called Bet land. One of the largest Bet land area of India is Ganga Kosi
Flood plain in Bihar.

www.iasbaba.com Page 31
IASbabas 60 DAY PLAN- UPSC PRELIMS 2017
ENVIRONMENT (Subject Wise Compilation)

Tila: Tila is the local name of soil mounds or small hillocks in North India.

Q.45) Which of the following statements are correct about Biological and Toxin weapons
Convention?

1. The convention entered into force in 1975.


2. It prohibits the development, production, and stockpiling of biological and toxin
weapons.
3. UN Biological Weapons Authority is the watchdog to check compliance of the
countries.

Select the code from below:

a) 1 and 2
b) 2 and 3
c) 1 and 3
d) All of the above

Q.45) Solution (a)

A draft of the BWC, submitted by the British was opened for signature on 10 April 1972 and
entered into force 26 March 1975 when twenty-two governments had deposited their
instruments of ratification. It commits the 178 states which are party to it as of December
2016 to prohibit the development, production, and stockpiling of biological and toxin
weapons. However, the absence of any formal verification regime to monitor compliance
has limited the effectiveness of the Convention.

Following Articles are present in the convention:

Article I: Never under any circumstances to acquire or retain biological weapons.


Article II: To destroy or divert to peaceful purposes biological weapons and
associated resources prior to joining.
Article III: Not to transfer, or in any way assist, encourage or induce anyone else to
acquire or retain biological weapons.
Article IV: To take any national measures necessary to implement the provisions of
the BWC domestically.
Article V: To consult bilaterally and multilaterally to solve any problems with the
implementation of the BWC.
Article VI: To request the UN Security Council to investigate alleged breaches of the
BWC and to comply with its subsequent decisions.

www.iasbaba.com Page 32
IASbabas 60 DAY PLAN- UPSC PRELIMS 2017
ENVIRONMENT (Subject Wise Compilation)

Article VII: To assist States which have been exposed to a danger as a result of a
violation of the BWC.
Article X: To do all of the above in a way that encourages the peaceful uses of
biological science and technology.

Q.46) The Minamata Convention is an international treaty designed to protect human


health and the environment from

a) Anthropogenic release of Mercury


b) Anthropogenic release of Heavy metals
c) Surface Ozone
d) Green house gases

Q.46) Solution (a)

The Minamata Convention on Mercury is an international treaty designed to protect human


health and the environment from anthropogenic emissions and releases of mercury and
mercury compounds. This Convention was a result of three years of meeting and
negotiating, after which the text of the Convention was approved by delegates representing
close to 140 countries on 19 January 2013 in Geneva and adopted and signed later that year
on 10 October 2013 on a Diplomatic Conference held in Kumamoto, Japan. The Convention
is named after the Japanese city Minamata. This naming is of symbolic importance as the
city went through devastating incident of mercury poisoning. It is expected that over the
next few decades, this international agreement will enhance the reduction of mercury
pollution from the targeted activities responsible for the major release of mercury to the
immediate environment.

Q.47) After a long gap of 84 years, Cobra lily has been rediscovered in NIlgiris. Which of
the following statements are correct about Cobra Lilies?

1. It is endemic specie found in Nilgiris only.


2. It was believed to be extinct in wild in the world.

Select the code from below:

a) 1 only
b) 2 only
c) Both 1 and 2
d) Neither 1 nor 2

Q.47) Solution (d)

www.iasbaba.com Page 33
IASbabas 60 DAY PLAN- UPSC PRELIMS 2017
ENVIRONMENT (Subject Wise Compilation)

Arisaema (Cobra Lily) is a large and diverse genus of the flowering plant family Araceae. The
largest concentration of species is in China and Japan, with other species native other parts
of southern Asia as well as eastern and central Africa, Mexico and eastern North America.

Of the handful cobra lily species found in the Nilgiris, only two are endemic.

http://www.thehindu.com/news/national/tamil-nadu/after-84-years-cobra-lily-blooms-
again/article18525894.ece

Q.48) Consider the following statements:

1. The National Green Tribunal is an executive body established in 2011 for quick
disposal of cases related to environment protection.
2. It is not bound by procedure laid down under the Civil Procedure Code, 1908, but
guided by the principles of Natural Justice.

Which of the above statements are correct?

a) 1 only
b) 2 only
c) Both 1 and 2
d) Neither 1 nor 2

Q.48) Solution (b)

The National Green Tribunal has been established on 18.10.2010 under the National Green
Tribunal Act 2010 for effective and expeditious disposal of cases relating to environmental
protection and conservation of forests and other natural resources including enforcement
of any legal right relating to environment and giving relief and compensation for damages to
persons and property and for matters connected therewith or incidental thereto. It is a
specialized body equipped with the necessary expertise to handle environmental disputes
involving multi-disciplinary issues. The Tribunal shall not be bound by the procedure laid
down under the Code of Civil Procedure, 1908, but shall be guided by principles of natural
justice.

The Tribunal's dedicated jurisdiction in environmental matters shall provide speedy


environmental justice and help reduce the burden of litigation in the higher courts. The
Tribunal is mandated to make and endeavour for disposal of applications or appeals finally
within 6 months of filing of the same. Initially, the NGT is proposed to be set up at five
places of sittings and will follow circuit procedure for making itself more accessible. New

www.iasbaba.com Page 34
IASbabas 60 DAY PLAN- UPSC PRELIMS 2017
ENVIRONMENT (Subject Wise Compilation)

Delhi is the Principal Place of Sitting of the Tribunal and Bhopal, Pune, Kolkata and Chennai
shall be the other 4 place of sitting of the Tribunal.

Q.49) National Water Policy is formulated by the Ministry of Water Resources of the
Government of India to govern the planning and development of water resources and
their optimum utilization. Which of the following statements is not correct about Indias
National Water Policy?

a) It treats water as an economic good to promote conservation and efficient use.


b) It ensures access to a minimum quantity of potable water for essential health and
hygiene to all citizens.
c) To keep aside a portion of the river flow to meet the ecological needs and to ensure
that the low and high flow releases correspond in time closely to the natural flow
regime.
d) It has set water allocation priorities to different sectors.

Q.49) Solution (d)

The main emphasis of National Water Policy 2012 is to treat water as economic good which
the ministry claims to promote its conservation and efficient use. This provision intended for
the privatization of water-delivery services is being criticized from various quarters.The
policy also does away with the priorities for water allocation mentioned in 1987 and 2002
versions of the policy. The policy was adopted with a disapproval from many states.

The other major features are:-

To ensure access to a minimum quantity of potable water for essential health and
hygiene to all citizens, available within easy reach of the household.
To curtail subsidy to agricultural electricity users.
Setting up of Water Regulatory Authority.
To keep aside a portion of the river flow to meet the ecological needs and to ensure
that the low and high flow releases correspond in time closely to the natural flow
regime.
To give statutory powers to Water Users Associations to maintain the distribution
system.
Project benefited families to bear part of the cost of resettlement & rehabilitation of
project affected families.
To remove the large disparity between stipulations for water supply in urban areas
and in rural areas.
To support a National Water Framework Law.

www.iasbaba.com Page 35
IASbabas 60 DAY PLAN- UPSC PRELIMS 2017
ENVIRONMENT (Subject Wise Compilation)

Q.50) Which of the following plants can be used as indicators of Sulphur Oxide pollution in
Air?

a) Ferns
b) Lichens
c) Liver Worts
d) Conifers

Q.50) Solution (b)

Sulfur dioxide is released if the fuel contains sulfur compounds. This gas contributes to acid
rain. Lichens can be used as air pollution indicators, especially of the concentration of sulfur
dioxide in the atmosphere. Lichens are plants that grow in exposed places such as rocks or
tree bark.

Q.51) Which of the following air pollution devices are suitable for removing dust particles
from air?

1. Cyclone separator
2. Electrostatic precipitator
3. Fabric filter
4. Wet scrubber

Select the code from below:

a) 1,2 and 3
b) 2,3 and 4
c) 1,3 and 4
d) All of the above

Q.51) Solution (d)

Cyclonic separation is a method of removing particulates from an air, gas or liquid stream.

An electrostatic precipitator (ESP) is a filtration device that removes fine particles, like dust
and smoke, from a flowing gas using the force of an induced electrostatic charge minimally
impeding the flow of gases through the unit.

Fabric Filters (also called baghouses and bagfilters) are air pollution control devices that
remove particulate matter from a gas stream by passing the dirty air through a layer of
cloth. The particulate matter deposits on the cloth surface and the clean air passes through.

www.iasbaba.com Page 36
IASbabas 60 DAY PLAN- UPSC PRELIMS 2017
ENVIRONMENT (Subject Wise Compilation)

Wet scrubbers are effective air pollution control devices for removing particles and/or gases
from industrial exhaust streams. A wet scrubber operates by introducing the dirty gas
stream with a scrubbing liquid typically water. Particulate or gases are collected in the
scrubbing liquid.

Q.52) The Government of India has put restrictions on the use of Loud Speakers/ public
address systems under The Noise pollution (Regulation and Control) Rule, 2000. Which of
the following statements are correct about regarding this?

1. A loud speaker or a public address system shall not be used except after obtaining
written permission from the authority.
2. A loud speaker or a public address system or any sound producing instrument or a
musical instrument or a sound amplifier shall not be used at night time except in
closed premises for communication within, like auditoria, conference rooms,
community halls, banquet halls or during a public emergency.
Select the code from below:

a) 1 only
b) 2 only
c) Both 1 and 2
d) Neither 1 nor 2

Q.52) Solution (c)

Restrictions on the use of loud speakers / public address system and sound producing
instruments.-
(1) A loud speaker or a public address system shall not be used except after obtaining
written permission from the authority.
(2) A loud speaker or a public address system or any sound producing instrument or a
musical instrument or a sound amplifier shall not be used at night time except in closed
premises for communication within, like auditoria, conference rooms, community halls,
banquet halls or during a public emergency.
(3) Notwithstanding anything contained in sub-rule (2), the State Government may subject
to such terms and conditions as are necessary to reduce noise pollution, permit use of loud
speakers or public address system and the like during night hours (between 10.00 p.m. to
12.00
midnight) on or during any cultural or religious festive occasion of a limited duration not
exceeding fifteen days in all during a calendar year. The concerned State Government shall

www.iasbaba.com Page 37
IASbabas 60 DAY PLAN- UPSC PRELIMS 2017
ENVIRONMENT (Subject Wise Compilation)

generally specify in advance, the number and particulars of the days on which such
exemption would be operative.
(4) The noise level at the boundary of the public place, where loudspeaker or public address
system or any other noise source is being used shall not exceed 10 dB (A) above the ambient
noise standards for the area or 75 dB (A) whichever is lower;
(5) The peripheral noise level of a privately owned sound system or a sound producing
instrument shall not, at the boundary of the private place, exceed by more than 5 dB (A) the
ambient noise standards specified for the area in which it is used.

Q.53) Which of the following traits should a waste material show to be categorised under
Hazardous waste?

1. Ignitability
2. Reactivity
3. Corrosivity
4. Toxicity

Select the code from below:

a) 1 and 2
b) 3 and 4
c) 1,3 and 4
d) All of the above

Q.53) Solution (d)

Characteristic hazardous wastes are materials that are known or tested to exhibit one or
more of the following four hazardous traits:

ignitability
reactivity
corrosivity
toxicity

Q.54) Which of the following is not a criterion for selecting a Ramsar Site?

a) If the site is representative, rare, or unique example of a natural or near-natural


wetland type found within the appropriate biogeographic region.
b) If it supports vulnerable, endangered, or critically endangered species or threatened
ecological communities.
c) If it supports plant and/or animal species at a critical stage in their life cycles, or
provides refuge during adverse conditions.

www.iasbaba.com Page 38
IASbabas 60 DAY PLAN- UPSC PRELIMS 2017
ENVIRONMENT (Subject Wise Compilation)

d) If it regularly supports 2,00,000 or more water-birds.

Q.54) Solution (d)

Criteria for Selecting a RAMSAR site:

Criterion 1:If the site is representative, rare, or unique example of a natural or near-
natural wetland type found within the appropriate biogeographic region.
Criterion 2: if it supports vulnerable, endangered, or critically endangered species or
threatened ecological communities.
Criterion 3: if it supports populations of plant and/or animal species important for
maintaining the biological diversity of a particular biogeographic region.
Criterion 4: if it supports plant and/or animal species at a critical stage in their life
cycles, or provides refuge during adverse conditions.
Criterion 5: if it regularly supports 20,000 or more water-birds.
Criterion 6: if it regularly supports 1% of the individuals in a population of one
species or subspecies of water-bird.
Criterion 7: if it supports a significant proportion of indigenous fish subspecies,
species or families, life-history stages, species interactions and/or populations that
are representative of wetland benefits and/or values and thereby contributes to
global biological diversity.
Criterion 8: if it is an important source of food for fishes, spawning ground, nursery
and/or migration path on which fish stocks, either within the wetland or elsewhere,
depend.
Criterion 9: if it regularly supports 1% of the individuals in a population of one
species or subspecies of wetland-dependent non-avian animal species.

Q.55) Convention on Biological Diversity entered into force in December 1993. Which of
the following were the objectives of CBD?

1. Conservation of Biological Diversity


2. Sustainable use of the components of Biological Diversity
3. The fair and equitable sharing of the benefits arising out of the utilization of genetic
resources
Select the code from below:

a) 1 and 2
b) 2 and 3
c) 1 only
d) All of the above

Q.55) Solution (d)

www.iasbaba.com Page 39
IASbabas 60 DAY PLAN- UPSC PRELIMS 2017
ENVIRONMENT (Subject Wise Compilation)

The Convention on Biological Diversity (CBD) entered into force on 29 December 1993. It
has 3 main objectives:

The conservation of biological diversity


The sustainable use of the components of biological diversity.
The fair and equitable sharing of the benefits arising out of the utilization of genetic
resources
The Convention was opened for signature on 5 June 1992 at the United Nations
Conference on Environment and Development (the Rio Earth Summit).
This convention is a legally binding framework treaty that has been ratified by180
countries.
The areas that are dealt by convention are conservation of biodiversity,sustainable
use of biological resources and equitable sharing of benefits arising fromtheir
sustainable use.
The convention came into force in 1993. Many biodiversity issuesare addressed
including habitat preservation, intellectual property rights, biosafety andindigenous
peoples rights.

Q.56) Consider the following statements:

1. World heritage Convention is under the aegis of UNESCO


2. Wild Life wing of the Ministry of Environment and Forests is associated with the
conservation of the Natural World Heritage sites.
3. Currently, six natural World Heritage Sites have been recognized by UNESCO in
India.

Which of the above statements is correct?

a) 1 only
b) 2 and 3
c) 1 and 3
d) All of the above

Q.56) Solution (d)

World Heritage Convention:

India is a member of World Heritage Convention responsible for listing of World Heritage
Sites, which include both Cultural and natural sites. The World Heritage Convention is a
Convention under the aegis of the United Nations Educational, Scientific and Cultural
Organization (UNESCO). Wild Life wing of the Ministry of Environment and Forests is
associated with the conservation of the Natural World Heritage sites.

www.iasbaba.com Page 40
IASbabas 60 DAY PLAN- UPSC PRELIMS 2017
ENVIRONMENT (Subject Wise Compilation)

Currently, six natural World Heritage Sites have been recognized by UNESCO in India, viz.,
Nanda Devi National Park, Kaziranga National Park, Manas National Park, Keoladeo National
Park, Sundarbans National Park. Apart from these, the Valley of Flowers National Park has
also been included in the list of World Heritage Sites as an extension of Nanda Devi National
Park.

Further, a serial cluster of following 39 sites from Western Ghats spread over 4 States have
also been inscribed as World Heritage Sites

Q.57) Haze has been a major urban problem of the recent time. To tackle this ASEAN
Agreement on transboundary haze pollution has been signed. Which of the following
statements are correct regarding this agreement?

1. The objective of the resolution is to reduce haze pollution in South and South East
Asia.
2. India has ratified the agreement.

Select the code from below:

a) 1 only
b) 2 only
c) Both 1 and 2
d) Neither 1 nor 2

Q.57) Solution (d)

ASEAN Agreement on Transboundary Haze Pollution


It is an environmental agreement signed in 2002 between all ASEAN nations to reduce haze
pollution in Southeast Asia.
As of June 2007, eight countries Viz. Malaysia, Singapore, Brunei, Darussalam, Myanmar ,
Vietnam , Thailand , Lao PDR , Cambodia have ratified the agreement.

Q.58) Consider the following statements regarding Saker Falcons:

1. Saker Falcons are found in semi arid to forest regions of Europe and Central Asia.
2. They migrate to South Asia and Africa during Winters.
3. They are listed as endangered Species in IUCN red list.

Which of the above statements are correct?

a) 1 and 2

www.iasbaba.com Page 41
IASbabas 60 DAY PLAN- UPSC PRELIMS 2017
ENVIRONMENT (Subject Wise Compilation)

b) 2 and 3
c) 1 and 3
d) All of the above

Q.58) Solution (d)

The current status of Saker Falcon has been changed from Vulnerable to Endangered this
year. Hence a question is probable.

Saker falcons (often simply called sakers) occur in the semi-desert and forest regions from
Eastern Europe to central Asia, where they are the dominant desert falcon. Saker falcons
migrate as far as northern parts of southern Asia and parts of Africa for the winter. Recently
(in 1997), sakers have been observed breeding as far west as Germany.

Kindly go through the following link to know more:

http://animaldiversity.org/accounts/Falco_cherrug/

Q.59) Consider the following statements regarding the Genetic Diversity:

1. Genetic diversity exists between different species and not within the same species.
2. It ensures that some species survive drastic changes and thus carry on the desirable
genes.
Which of the above statements are correct?

a) 1 only
b) 2 only
c) Both 1 and 2
d) Neither 1 nor 2

Q.59) Solution (b)

Genetic Diversity
It is variation in genes within a particular species.
A single species might show high diversity at the genetic level [E.g. Man: Chinese, Indian
American, African etc.]. India has more than 50,000 genetically different strains of rice, and
1,000 varieties of mango.
It is the total number of genetic characteristics in the genetic makeup of a species.
It allows species to adapt to changing environment.
It ensures that some species survive drastic changes and thus carry on the desirable genes
(just like what they say in movie Lucy, carrying information).

www.iasbaba.com Page 42
IASbabas 60 DAY PLAN- UPSC PRELIMS 2017
ENVIRONMENT (Subject Wise Compilation)

The beautiful butterflies, roses, parakeets or coral in a myriad hue, shapes and sizes are
result of Biodiversity.

Q.60) Consider the following statements:

1. Endemic species are species unique to a defined geographical location.


2. Endemic species are least prone to extinction due to habitat loss.
3. Among animals, insects are the most species rich taxonomical group.

Which of the above statements are correct?

a) 1 only
b) 2 and 3
c) 1 and 3
d) All of the above

Q.60) Solution (c)

Species Diversity
It is the variety of living organisms on earth.
Species have different genes and they do not inter breed in nature.
Closely - related species have many common hereditary characteristics. O For example -
about 98.4% of the genes of humans and chimpanzees are the same.

Species Diversity is given by the formula: One specie population/total number of organisms
across all species. (In a given biome.)
Zero means infinite diversity and one represents only one species present.
Endemism it is the ecological state of a species being unique to a defined geographic
location, such as an island, nation, country or other defined zone, or habitat type.
Organisms that are indigenous to a place are not endemic to it if they are also found
elsewhere. A particular type of animal or plant may be endemic to a zone, a state or a
country. The extreme opposite of endemism is cosmopolitan distribution.
Endemic species are most prone to extinction due to habitat loss.
Among animals, insects are the most species rich taxonomic group, making up more
than 70 per cent of the total. That means, out of every 10 animals on this planet, 7 are
insects.

Q.61) Which of the following statements correctly explains the term Bioprospecting?

a) Finding a new habitat for a vulnerable specie.


b) Artificially mating two different species to get new species.

www.iasbaba.com Page 43
IASbabas 60 DAY PLAN- UPSC PRELIMS 2017
ENVIRONMENT (Subject Wise Compilation)

c) The search for economically valuable genetic and biochemical resources from
nature.
d) The search for alternate keystone specie for an ecosystem.

Q.61) Solution (c)

Bioprospecting
Bioprospecting can be defined as the systematic search for and development of new
sources of chemical compounds, genes, micro organisms, macro organisms, and other
valuable products from nature. It entails the search for economically valuable genetic and
biochemical resources from nature. So, in brief, bioprospecting means looking for ways to
commercialize biodiversity.
Nations endowed with rich biodiversity explore molecular, genetic and species level
diversity to derive products of economic importance.

Q.62) According to Indian Forest Report 2015, approximately how much land in India is
under forest cover?

a) 35%
b) 15%
c) 24%
d) 18%

Q.62) Solution (c)

Indias geographical area under forest and tree cover 24.16% (Indianforest report 2015).
Of the 34 globally identified biodiversity hotspots, India harbours two hotspots, i.e., Eastern
Himalayas, Western Ghats and Sri Lanka

Q.63) Which of the following statements are correct about biogeographic Realms?

1. Realm is a continent or sub - continent sized area with unifying features of


geography and fauna & flora.
2. The world has been divided into eight biogeographic realms.
3. Neotropical realm covers Central and South America.

Select the code from below:

a) 1 and 2
b) 2 and 3
c) 1 and 3
d) All of the above

www.iasbaba.com Page 44
IASbabas 60 DAY PLAN- UPSC PRELIMS 2017
ENVIRONMENT (Subject Wise Compilation)

Q.63) Solution (d)

Realms
Biogeographic realms are large spatial regions within which ecosystems share a broadly
similar biological evolutionary history.

Realm is a continent or sub continent sized area with unifying features of geography and
fauna & flora.
In world - 8 biogeographic realms are there
Nearctic realm
Palaearctic realm
Africotropical realm
Indomalayan realm
Ocenaia realm
Australian realm
Antarctic realm
Neotropical realm

Q.64) Which of the following are the examples of Arachnids?

1. Mosquito
2. Tick
3. Spider

www.iasbaba.com Page 45
IASbabas 60 DAY PLAN- UPSC PRELIMS 2017
ENVIRONMENT (Subject Wise Compilation)

4. Scorpion

Select the code from below:

a) 1,2 and 3
b) 2,3 and 4
c) 1,3 and 4
d) All of the above

Q.64) Solution (b)

Arachnids

They are a class of joint legged in vertebrate animals (arthropods).


Example spiders, scorpions, ticks and mites.
They do not have antennae.
They have 2 body parts and 4 pairs of legs.

Q.65) Which of the following are correct differences between angiosperms and
gymnospers?

1. Angiosperms have seeds while gymnosperms do not.

www.iasbaba.com Page 46
IASbabas 60 DAY PLAN- UPSC PRELIMS 2017
ENVIRONMENT (Subject Wise Compilation)

2. Angiosperms bear fruits while gymnosperms do not.

Select the code from following:

a) 1 only
b) 2 only
c) Both 1 and 2
d) Neither 1 nor 2

Q.65) Solution (b)

Gymnosperms (gymnos=naked, sperma = seed) are the naked seeded plants.


Their naked condition stands in contrast to the seeds and ovules of flowering plants
(angiosperms), which are enclosed within an ovary.
They have very simple flowers without accessory whorls and the microsporophylls
(stamens) and carpels remain aggregated in cones.
Ovules are present on the surface of the carpels and are directly pollinated by the pollen
grains.
There is nothing like ovary, style and stigma, and naturally there is no fruit.
E.g. Cycas, Pinus, Gnetum.

Like gymnosperms, angiosperms are seed producing plants.


They are distinguished from gymnosperms by characteristics including flowers, endosperm
within the seeds, and the production of fruits that contain the seeds.
Angiosperms are the most highly developed plants which bear flowers having conspicuous
accessory and essential whorls.
O Carpels have the ovary, style and stigma.

Q.66) Which of the following is the correct criterion to keep a specie in Critically
endangered list of IUCN red data book?

1. Reduction of population > 50% in last 10 years.


2. Reduction in population size (Less than 50 mature individuals).

Select the code from below:

a) 1 only
b) 2 only
c) Both 1 and 2
d) Neither 1 nor 2

Q.66) Solution (b)

www.iasbaba.com Page 47
IASbabas 60 DAY PLAN- UPSC PRELIMS 2017
ENVIRONMENT (Subject Wise Compilation)

Critically Endangered (CR)


Extremely high risk of extinction in the wild.
A taxon is Critically Endangered when the best available evidence indicates that it
meets any of the criteria for Critically Endangered.

Criteria
Reduction in population (> 90% over the last 10 years),
Reduction in Population size (number less than 50 mature individuals)
Quantitative analysis showing the probability of extinction in wild in at least 50% in
their 10 years)
It is therefore considered to be facing an extremely high risk of extinction in the
wild.

Q.67) Which of the following statements regarding Namdhapa Flying Squirrel are correct?

1. It is found only in Uttarakhand state of India.


2. Its status is critically Endangered in IUCN red list.
3. It is hunted for food.

Select the code from below:

a) 1, 2 and 3
b) 1 and 2
c) 2 and 3
d) 1 and 3

Q.67) Solution (c)

The Namdapha Flying Squirrel

It is a unique (the only one in its genus) flying squirrel that is restricted to a single
valley in the Namdapha National park or wild life sanctuary in Arunachal Pradesh.
Habitat - Tropical forest.
Distribution - Found only in Namdapha Tiger Reserve in Arunachal Pradesh.
Threats - Hunted for food.

Q.68) Which of the following statements are correct about the Primary criteria for
selection of Biosphere Reserves in India?

1. A site that must contain an effectively protected and minimally disturbed core area.
2. It should include Additional land and water suitable for research.

www.iasbaba.com Page 48
IASbabas 60 DAY PLAN- UPSC PRELIMS 2017
ENVIRONMENT (Subject Wise Compilation)

3. Core area should be typical of a biogeographical unit and large enough to sustain
viable populations representing all tropic levels in the ecosystem.
Select the code from below:

a) 1,2 and 3
b) 1 and 2
c) 2 and 3
d) 1 and 3

Q.68) Solution (a)

Criteria for selection of BRs


Primary criteria
A site that must contain an effectively protected and minimally disturbed core area.
It should include
O Additional land and water suitable for research.
O Demonstration of sustainable methods of research and management.
Core area should be typical of a biogeographical unit and large enough to sustain viable
populations representing all tropic levels in the ecosystem.

Secondary criteria
Areas having rare and endangered species.
Areas having diversity of soil and micro climatic conditions and indigenous varieties of
biota.
Areas potential for preservation of traditional tribal or rural modes of living for
harmonious use of environment.

Q.69) Which of the following can be threats to the biodiversity of a geographical area?
1. Global warming
2. Fragmentation of habitat
3. Invasion of alien species
4. Promotion of vegetarianism
Select the correct answer using the codes given below:
a) 1, 2 and 3 only
b) 2 and 3 only
c) 1 and 4 only
d) 1, 2, 3 and 4

Q.69) Solution (d)

www.iasbaba.com Page 49
IASbabas 60 DAY PLAN- UPSC PRELIMS 2017
ENVIRONMENT (Subject Wise Compilation)

The question was asked by UPSC in 2012.

First three points are straight forward.

Promotion of Vegetarianism would mean more demand of food grains and vegetables that
will require more agricultural land hence clearing more forest and loss of habitat. This is the
reason why alternate sources of food is being promoted by the government with different
colourful revolutions like red revolution, pink revolution and blue revolution, to reduce
pressure on agricultural land.

Q.70) Global warming potential (GWP) describes the impact of each gas on global
warming. Which of the following gases has the highest GWP?

a) Methane
b) Sulphur hexafluoride
c) CFCs
d) HCFCs

Q.70) Solution (b)

Global warming Potential (GWP)


Global warming potential describes the impact of each gas on global warming.
Two most important characteristics of a GHG
O How well the gas absorbs energy
O How long the gas stays in the atmosphere?
The Global Warming Potential (GWP) for a gas is a measure of the total energy that a gas
absorbs over a particular period of time (usually 100 years), compared to carbon dioxide.
Higher GWP gases absorb more energy Contribute more to warming earth.
Carbon dioxide (C02) has a GWP of 1 and serves as a baseline for other GWP values.
The larger the GWP, the more warming the gas causes.
O E.g.methane's 100 year GWP is 21, which means that methane will cause 21 times as
much warming as an equivalent mass of CO2 over a 100 year time period.

www.iasbaba.com Page 50
IASbabas 60 DAY PLAN- UPSC PRELIMS 2017
ENVIRONMENT (Subject Wise Compilation)

Q.71) Which of the following statements regarding Ocean Acidification are correct?

1. The pH of Oceans in certain areas have gone down less than 7.


2. There is an ongoing decrease in the pH of our oceans because of higher uptake of
CO2 from the atmosphere.

Select the code from below:

a) 1 only
b) 2 only
c) Both 1 and 2
d) Neither 1 nor 2

Q.71) Solution (b)

Ocean acidification is the ongoing decrease in the pH of the Earth's oceans, caused by the
uptake of carbon dioxide (CO2) from the atmosphere.
It increases the concentration of hydrogen ions and decreases the concentration of
carbonate ions.
Seawater is slightly basic (meaning pH > 7), and the process in question is a shift towards pH
neutral conditions rather than a transition to acidic conditions (pH < 7).

Q.72) Consider the following statements regarding IPCC:

www.iasbaba.com Page 51
IASbabas 60 DAY PLAN- UPSC PRELIMS 2017
ENVIRONMENT (Subject Wise Compilation)

1. It has been established by United Nations Environment Program (UNEP) and World
Meteorological Organisation.
2. It does not conduct any research nor does it monitor climate related data or
parameters.
Which of the above statements are correct?

a) 1 only
b) 2 only
c) Both 1 and 2
d) Neither 1 nor 2

Q.72) Solution (c)

The UN General Assembly adopted a resolution, in 1988, on the subject and endorsed the
UNEP/WMO proposal for the setting up of the Inter Governmental Panel on Climate
Change (IPCC).
Established by - United Nations Environment Programme (UNEP) and the World
meteorological Organization (WMO)
The IPCC is a scientific body. It reviews and assesses the most recent scientific, technical
and socio economic information produced worldwide relevant to the understanding of
climate change.
It is open to all member countries of the UN and WMO. Currently 195 countries are
members of IPCC.
It does not conduct any research nor does it monitor climate related data or parameters.
Review is an essential part of the IPCC process, to ensure an objective and complete
assessment of current information.
By endorsing the IPCC reports, governments acknowledge the authority of their scientific
content. The work of the organization is therefore policy relevant and yet policy neutral,
never policy prescriptive.
It has also responded to the need of the UNFCCC for information on scientific and
technical matters through Special Reports, Technical Papers and Methodology Reports.

Q.73) Recently Forest and Environment Department of an Indian state learnt tricks from
Mankidia tribe to trap monkeys. Where are these tribes found?

a) Western Ghats of Karnataka


b) Odisha
c) Rajasthan
d) Uttar Pradesh

www.iasbaba.com Page 52
IASbabas 60 DAY PLAN- UPSC PRELIMS 2017
ENVIRONMENT (Subject Wise Compilation)

Q.73) Solution (b)

Faced with rising incidents of monkeys raiding vegetable gardens, invading homes and
attacking villagers in coastal Odisha, the State's Forest and Environment Department has
asked its personnel to learn a few tricks from the Mankidia tribe to contain the menace.

Upon spotting a monkey in a tree, the tribes surround the monkey by climbing nearby trees
and structures. Once it is surrounded, they shake the tree till the animal falls. A team on the
ground, who are ready with a net, quickly trap the animal

Source: http://www.thehindu.com/news/cities/Delhi/odisha-forest-staff-learn-how-to-trap-
monkeys-from-mankidia-tribesmen/article18349053.ece

Q.74) The rate of biomass production is called productivity. It is expressed in terms of

a) kcal m2
b) g 2
c) g2 yr1
d) None of the above

Q.74 Solution (c)

A constant input of solar energy is the basic requirement for any ecosystem to function and
sustain. Primary production is defined as the amount of biomass or organic matter
produced per unit area over a time period by plants during photosynthesis. It is expressed in
terms of weight (g 2) or energy (kcal m2). The rate of biomass production is called
productivity. It is expressed in terms of g2 yr1 or (kcal m2) yr1 to compare the
productivity of different ecosystems. It can be divided into gross primary productivity (GPP)
and net primary productivity (NPP). Gross primary productivity of an ecosystem is the rate
of production of organic matter during photosynthesis. A considerable amount of GPP is
utilised by plants in respiration. Gross primary productivity minus respiration losses (R), is
the net primary productivity (NPP).

Q.75) Consider the following about Ecological Pyramids:

1. It takes into account the same species belonging to two or more trophic levels
2. Saprophytes plays a vital role in ecological pyramids
3. Pyramid of energy is the only pyramid which is always upright

www.iasbaba.com Page 53
IASbabas 60 DAY PLAN- UPSC PRELIMS 2017
ENVIRONMENT (Subject Wise Compilation)

Select the incorrect statement/s

a) 1 Only
b) 1 and 2
c) 2 only
d) None

Q.75) Solution (b)

The base of each pyramid represents the producers or the first trophic level. Apex
represents tertiary or top level consumers. In general, all pyramids are upright, but there
are few exceptions.

There are three ecological pyramids that are usually studies:

1. Pyramid of Number:

It represents the total number of organisms at each trophic level. It is always upright but in
a tree ecosystem pyramid of number is inverted.

2. Pyramid of Biomass:

It represents total weight of the organisms in each trophic level.

It can be:

(i) Upright, e.g., in grasslands.

(ii) Inverted, e.g., in pond ecosystem.

www.iasbaba.com Page 54
IASbabas 60 DAY PLAN- UPSC PRELIMS 2017
ENVIRONMENT (Subject Wise Compilation)

3. Pyramid of Energy:

It represents total energy of the organisms in each trophic level. Pyramid of energy is always
upright, i.e., it can never be inverted, because when energy is transferred from a particular
trophic level to the next trophic level some energy is always lost as heat at each step.

Some important points about ecological pyramids are given under:

(i) A given organism may occupy more than one trophic level simultaneously.

(ii) Trophic level represents a functional level.

(iii) A given species may occupy more than one trophic level in the same ecosystem at the
same time.

For example, a sparrow is primary consumer, when it eats seeds, fruit, peas, etc., and a
secondary consumer when it eats insects and worms.

(iv) In most ecosystems, all the pyramids of number, biomass, energy are upright, i.e.,
producers are more in number and biomass than the herbivores and herbivores are more in
number and biomass than carnivores.

(v) Also energy at lower trophic level is always more at higher trophic level. However, there
are exceptions to this generalisation.

www.iasbaba.com Page 55
IASbabas 60 DAY PLAN- UPSC PRELIMS 2017
ENVIRONMENT (Subject Wise Compilation)

(vi) Pyramid of biomass in sea is inverted because the biomass of fishes far exceeds that of
phytoplankton.

(vii) Each bar in the energy pyramid indicates the amount of energy present at each trophic
level in a given time or annually per unit area.

Limitations of Ecological Pyramids:

(i) It never takes into account the same species belonging to two or more trophic levels.

(ii) It assumes a simple food chain, something that almost never exists in nature.

(iii) It does not accommodate a food web.

(iv) Saprophytes are not given any place in ecological pyramids even though; they play an
important role in ecosystem.

Q.76) Consider the statements regarding Carbon and Phosphorous Cycle:

1. Like carbon cycle, there is respiratory release of phosphorus into atmosphere


2. Atmospheric inputs of phosphorus through rainfall are much greater than carbon
inputs
3. Gaseous exchanges of phosphorus between organism and environment are
negligible unlike carbon

Select the incorrect statement/s

a) Only 2
b) 2 and 3
c) 1 and 2
d) Only 1

Q.76) Solution (c)

Unlike carbon cycle, there is no respiratory release of phosphorus into atmosphere: Focus
on basic concept (How can phosphorous be released through respiration?)

Atmospheric inputs of phosphorus through rainfall are much smaller than carbon inputs:

www.iasbaba.com Page 56
IASbabas 60 DAY PLAN- UPSC PRELIMS 2017
ENVIRONMENT (Subject Wise Compilation)

Gaseous exchanges of phosphorus between organism and environment are negligible unlike
carbon

Basic Info:

Nutrients are basically required by all organisms for the overall growth,
Hint development, reproduction and to carry out various life processes. The interesting fact is
that the nutrient amount is fixed in the environment. Thus, the nutrients are naturally
recycled for the constant supply to the organisms. This movement of the nutrients through
various components of the ecosystem is called nutrient cycling. Broadly, we have two types
of nutrient cycling in nature:

Gaseous cycling This kind of cycling takes place for the gases and the atmosphere acts as
the reservoir for these gases i.e Carbon

Sedimentary cycling This kind of cycling takes place for the nutrients that exist in their
elemental form and the earths crust acts as the reservoir i.e Phosphorous

Q.77) Consider the following statements regarding Flowering and Non-flowering plants-

1. Both flowering and non-flowering plants have seeds


2. Flowering plants are known as gymnosperms while non-flowerings are known as
angiosperms
3. Examples of angiosperms includes, grass and bananas while pine and conifers are
examples of gymnosperms

Select the correct statement/s from above

a) 1 and 2
b) 2 and 3
c) Only 3
d) 1 and 3

Q.77) Solution (d)

Flower is the reproductive organ of the plant. Flowers can be composite or simple. Anyway
using flower as the classification criteria makes this a natural classification.

Flowering plants are also known as Angiosperms.

www.iasbaba.com Page 57
IASbabas 60 DAY PLAN- UPSC PRELIMS 2017
ENVIRONMENT (Subject Wise Compilation)

Angiosperms

Angiosperms are plants that


are seed-bearing. Their
reproductive structures are
flowers were the ovules are
enclosed in an ovary.
Angiosperms can found in
most habitats from forests,
grasslands, sea margins and
even deserts. The leaves of angiosperms are flat. Examples of Angiosperms are grass,
Apples, Oranges and Bananas.

Angiosperms are plants that have flowers and their reproduction is done by flowers.
All flowering plants produce seeds and it is covered by a fruit.
Angiosperms have complicated flowers with complicated structure.
These are the most developed plants in the world.

There are 2 types of Angiosperms namely Dicotyledonous Plants and Monocotyledonous


plants.

1. Dicotyledonous Plants

Dicotyledonous flowering plants are the widely expanded around the world. The seed of this
type of plants have 2 cotyledons inside it. Also the root system has a long tap root. The
leaves have webbed veins and the flowers have 5 or multiple of 5 petals.

Examples for Dicotyledonous plants

Jak
Mango
Lemon
Apple
Tomato

2. Monocotyledonous Plants

Monocotyledonous flowering plants are very few when compared to dicotyledonous plants.
These plants dont have a tap root system, but a fibrous root system. The seed has only 1
cotyledons and the veins of the leaves are parallel. Number of petals in the flowers are 3 or
multiples of 3.

www.iasbaba.com Page 58
IASbabas 60 DAY PLAN- UPSC PRELIMS 2017
ENVIRONMENT (Subject Wise Compilation)

Examples for Monocotyledonous plants

Coconut
Wheat
Palm
Rice
Garlic

Non-Flowering Plants

Non-flowering plants do have reproductive organs available in the flowers, but these are not
complicated as Dicotyledonous flowers. We can further classify non-flowering plants as with
seeds and without seeds.

Gymnosperms

Gymnosperms are plant with seeds and the embryo is inside a seed. The seed is not covered
(not enclosed) in a fruit. These are the less advanced type of seed plants.

Examples for Gymnosperms plants

Yews
Ginkgo
Cycads
Conifers
Pine

Bryophyta

Bryophyta doesnt have seeds. There are so many species under this type and body of the
plant may be a thallus. Sometimes they do have leaves. There are no true roots, but roots
like structures available. These are also known as Rhizoids and grow on moist walls and
ground.

Hint: India Year Book: Environment

Q.78) Consider the following regarding Zoological Survey of India (ZSI)

1. It publishes Red Data Book on Indian Animals

www.iasbaba.com Page 59
IASbabas 60 DAY PLAN- UPSC PRELIMS 2017
ENVIRONMENT (Subject Wise Compilation)

2. The Zoological Survey of India (ZSI) is the premier taxonomic research organization in
India
3. It was established after the independence of India

Select the correct statement/s

a) 1 and 2
b) 2 and 3
c) Only 2
d) 1, 2 and 3

Q.78) Solution (a)

The Zoological Survey of India (ZSI) was established on 1st July, 1916 to promote survey,
exploration and research leading to the advancement in our knowledge of various aspects
of exceptionally rich life of the erstwhile British Indian Empire . The survey has its genesis
in the establishment of the Zoological Section of the Indian Museum at Calcutta in 1875. By
gradually strengthening its staff and expanding its research program, the Survey has met the
challenge of the past and is on its way to meet the demands of the future.

Primary objectives

Exploring, Surveying, Inventorying and Monitoring of faunal diversity in various


states, selected ecosystems and protected areas of India.
Taxonomic studies of the faunal components collected.
Status survey of Threatened and Endemic species.
Preparation of Red Data Book, Fauna of India and Fauna of States.
Bio-ecological studies on important communities/species.
Preparation of database for the recorded species of the country.
Maintenance and Development of National Zoological Collections.
Training, Capacity Building and Human Resource Development.
Faunal Identification, Advisory services and Library Services.
Publication of results including Fauna of India, Fauna of States and Fauna of
Conservation Areas.

Secondary objectives

GIS and Remote Sensing studies on recorded animal diversity as well as on


threatened species.
Chromosomal Mapping and DNA Barcoding.
Environmental Impact Studies.

www.iasbaba.com Page 60
IASbabas 60 DAY PLAN- UPSC PRELIMS 2017
ENVIRONMENT (Subject Wise Compilation)

Maintenance of Musea at Headquarters and Regional Centres.


Development of ENVIS and CITES Centers.
Research Fellowship, Associateship and Emeritus Scientists Programme.
Collaborative research programmes on Biodiversity with other Organizations in India
and abroad.

Q.79) Consider the following w.r.t Animal Welfare Board of India (AWBI)

1. The Animal Welfare Board of India is a non-statutory advisory body on Animal


Welfare Laws and promotes animal welfare in the country.
2. It was established under the provisions of Wildlife Protection Act, 1972
3. World Animal Day is celebrated on 4th October

Select the incorrect statements

a) 1 and 2
b) 1 and 3
c) 2 and 3
d) 1, 2 and 3

Q.79) Solution (a)

The Animal Welfare Board of India is a statutory advisory body on Animal Welfare Laws and
promotes animal welfare in the country. Established in 1962 under Section 4 of the
Prevention of Cruelty to Animals Act, 1960.

World Animal Day - 4 October

MISSION: TO RAISE THE STATUS OF ANIMALS IN ORDER TO IMPROVE WELFARE STANDARDS


AROUND THE GLOBE

Q.80) Consider the following regarding Wildlife Protection Act, 1972

1. This was the first umbrella act which established schedules of protected plant and
animal species.
2. The act has six schedules that extends to whole of India
3. It has given criterions to declare national parks and wildlife sanctuaries

Select the correct statement/s

www.iasbaba.com Page 61
IASbabas 60 DAY PLAN- UPSC PRELIMS 2017
ENVIRONMENT (Subject Wise Compilation)

a) 1 and 2
b) 1 and 3
c) 2 and 3
d) 1, 2 and 3

Q.80) Solution (b)

The Wildlife Protection Act, 1972 is an Act of the Parliament of India enacted for protection
of plants and animal species. Before 1972, India only had five designated national parks.
This was the first umbrella act which established schedules of protected plant and animal
species. By this act, hunting or harvesting these species was largely outlawed.
Among other reforms, the Act established schedules of protected plant and animal species;
hunting or harvesting these species was largely outlawed.

The Act provides for the protection of wild animals, birds and plants; and for matters
connected therewith or ancillary or incidental thereto. It extends to the whole of India,
except the State of Jammu and Kashmir which has its own wildlife act. It has six schedules
which give varying degrees of protection. Schedule I and part II of Schedule II provide
absolute protection - offences under these are prescribed the highest penalties. Species
listed in Schedule III and Schedule IV are also protected, but the penalties are much
lower. Schedule V includes the animals which may be hunted. The plants in Schedule VI are
prohibited from cultivation and planting. The hunting to the Enforcement authorities have
the power to compound offences under this Schedule (i.e. they impose fines on the
offenders). Up to April 2010 there have been 16 convictions under this act relating to the
death of tigers.

Q.81) Consider the following statements:

1. Native rocks predominantly contribute nitrogen to plants or animals


2. Nitrogen fixation in aquatic ecosystems is done by free-living microbes
3. Nitrogen fixation in terrestrial ecosystem is carried out by symbiotic microbes

Which of the statements given above is/are correct?

a) 1 and 2
b) 2 and 3
c) Only 3
d) 1, 2 and 3

www.iasbaba.com Page 62
IASbabas 60 DAY PLAN- UPSC PRELIMS 2017
ENVIRONMENT (Subject Wise Compilation)

Q.81) Solution (b)

Remember that, the ultimate source of nitrogen to the ecosystem is molecular nitrogen in
the atmosphere which cannot be directly metabolized by plants or animals.

Native rocks do not contain nitrogen hence do not contribute nitrogen to plants or animals.
And nitrogen fixation in terrestrial ecosystem is predominantly carried out by symbiotic
microbes, whereas bulk of fixation in aquatic ecosystems is done by free-living microbes.

Q.82) Consider the following statements

1. Tropical rain forests soils are poorly leached


2. Tropical rain forest soils contain less organic matter than temperate forests
3. Tropical rain forests have very thick topsoil

Which of the statements given above is/are correct?

a) 1 and 2
b) Only 2
c) 2 and 3
d) None of the above

Q.82) Solution (b)

Did you know that tropical soils are among the deepest in the world! Many tropical soils
have been under forest cover for millions of years. Over this period, and under high rainfall
conditions, deep tropical soils have been formed from the underlying rock.

Tropical soils are often several metres deep, but the soils are often washed out, or
strongly leached, with large amounts of nutrients and minerals being removed from the
subsoils and considerable thickness of rock broken down to produce soil. Over many
millions of years this leaching has left most of the soils lacking many of the fundamental
nutrients needed by the above ground vegetation.

So how does such a lush vegetation exist if the soils are so depleted of nutrients? The
answer lies in the very thin topsoils, made up mainly of decaying vegetal and animal
remains. An amazing cycle exists between the huge body of vegetation above ground and
this thin topsoil. The rainforest depends for its nutrients on the constant recycling of its
enormous biomass.

www.iasbaba.com Page 63
IASbabas 60 DAY PLAN- UPSC PRELIMS 2017
ENVIRONMENT (Subject Wise Compilation)

Millions of years of weathering and torrential rains have washed most of the nutrients out
of the soil. More recent volcanic soils, however, can be very fertile. Tropical rain forest soils
contain less organic matter than temperate forests and most of the available nutrients are
found in the living plant and animal material. Nutrients in the soil are often in forms that are
not accessible by plants.

Q.83) Consider the following statements regarding Tropic Levels

1. The respiration cost decreases along the successive higher tropic levels.
2. The length of food chains in an ecosystem is limited because of 10% law.

Select the correct statement/s

a) 1 Only
b) 2 Only
c) Both
d) None

Q.83) Solution (b)

Respiration cost will increase sharply along successive higher tropic levels.

The efficiency with which energy or biomass is transferred from one trophic level to the next
is called the ecological efficiency.

Consumers at each level convert on average only about 10% of the chemical energy in their
food to their own organic tissue (the ten-percent law). For this reason, food chains rarely
extend for more than 5 or 6 levels.

At the lowest trophic level (the bottom of the food chain), plants convert about 1% of the
sunlight they receive into chemical energy. It follows from this that the total energy
originally present in the incident sunlight that is finally embodied in a tertiary consumer is
about 0.001%.

Q.84) Consider the following

1. Primary succession is faster than Secondary succession


2. Pioneer species are required in secondary succession

Which of the given statements is/are true?

www.iasbaba.com Page 64
IASbabas 60 DAY PLAN- UPSC PRELIMS 2017
ENVIRONMENT (Subject Wise Compilation)

a) 1 Only
b) 2 Only
c) Both
d) None

Q.84) Solution (d)

Ecological succession is the gradual process by which ecosystems change and develop over
time. Nothing remains the same and habitats are constantly changing. There are two main
types of succession, primary and secondary.

Primary succession is the series of community changes which occur on an entirely new
habitat which has never been colonized before. For example, a newly quarried rock face or
sand dunes.

Secondary succession is the series of community changes which take place on a previously
colonized, but disturbed or damaged habitat. For example, after felling trees in a woodland,
land clearance or a fire.

Simply put, secondary succession is the ecological succession that occurs after the initial
succession has been disrupted and some plants and animals still exist. It is usually faster
than primary succession as:

Soil is already present, so there is no need for pioneer species;


Seeds, roots and underground vegetative organs of plants may still survive in the
soil.

Q.85) Which of the following are ectoparasites?

1. Protozoa
2. Mites
3. Lice
4. Ticks

Select from the given code:

a) 1, 2 and 3
b) 2, 3 and 4
c) 1, 3 and 4
d) All of the above

www.iasbaba.com Page 65
IASbabas 60 DAY PLAN- UPSC PRELIMS 2017
ENVIRONMENT (Subject Wise Compilation)

Q.85) Solution (b)

Parasitism is a non-mutual relationship between species, where one species, the parasite,
benefits at the expense of the other, the host.

Parasites are classified based on their interactions with their hosts and on their life cycles.
An obligate parasite is totally dependent on the host to complete its life cycle, while
a facultative parasite is not. A direct parasite has only one host while an indirect parasite
has multiple hosts. For indirect parasites, there will always be a definitive host and an
intermediate host.

Parasites that live on the outside of the host, either on the skin or the outgrowths of the
skin, are called ectoparasites (e.g. Ticks, lice, fleas, and some mites).

Those that live inside the host are called endoparasites (including all parasitic worms).
Endoparasites can exist in one of two forms: intercellular parasites (inhabiting spaces in the
host's body) or intracellular parasites (inhabiting cells in the host's body). Intracellular
parasites, such as protozoa, bacteria or viruses, tend to rely on a third organism, which is
generally known as the carrier or vector. The vector does the job of transmitting them to the
host. An example of this interaction is the transmission of malaria, caused by a protozoan of
the genus Plasmodium, to humans by the bite of an anopheline mosquito.

Those parasites living in an intermediate position, being half-ectoparasites and half-


endoparasites, are called mesoparasites.

An epiparasite is one that feeds on another parasite. This relationship is also sometimes
referred to as hyperparasitism, exemplified by a protozoan (the hyperparasite) living in the
digestive tract of a flea living on a dog.

Social parasites take advantage of interactions between members of social organisms such
as ants, termites, and bumblebees.

Q.86) Consider the following statements

1. Mutualisms is exemplified by the nitrogen fixing bacteria


2. Algae and fungi exhibit mutually beneficial relationship in lichens

Which of the statements given above is/are correct?

a) Only 1
b) Only 2

www.iasbaba.com Page 66
IASbabas 60 DAY PLAN- UPSC PRELIMS 2017
ENVIRONMENT (Subject Wise Compilation)

c) Both
d) None

Q.86) Solution (c)

Symbiotic nitrogen fixation occurs in plants that harbor nitrogen-fixing bacteria within their
tissues. The best-studied example is the association between legumes and bacteria in the
genus Rhizobium.

Each of these is able to survive independently (soil nitrates must then be available to the
legume), but life together is clearly beneficial to both. Only together can nitrogen fixation
take place. A symbiotic relationship in which both partners benefits is called mutualism.

The fungi provide protection to algae and the algae prepares food for the fungi, thereby
exhibiting mutually beneficial relationship.

Q.87) Consider the following statements

1. The interaction where one species is benefitted and the other is neither benefitted
nor harmed is called amensalism
2. In commensalism on the other hand one species is harmed whereas the other is
unaffected.

Which of the statements given above is/are correct?

a) Only 1
b) Only 2
c) Both
d) None

Q.87) Solution (d)

The statements are interchanged. It is vice-versa

Q.88) Consider the statements regarding Aquatic Biome

1. The topmost zone near the shore of a lake or pond is the littoral zone.
2. The near-surface open water surrounded by the littoral zone is the profundal zone.

www.iasbaba.com Page 67
IASbabas 60 DAY PLAN- UPSC PRELIMS 2017
ENVIRONMENT (Subject Wise Compilation)

3. The deep-water part of the lake or pond is the limnetic zone.

Which of the statements given above is/are correct?

a) 1 and 2
b) 1 Only
c) 2 and 3
d) 3 Only

Q.88) Solution (b)

The aquatic biome can be broken down into two basic regions, freshwater (i.e, ponds and
rivers) and marine (i.e, oceans and estuaries).

Freshwater Regions

Freshwater is defined as having a low salt concentrationusually less than 1%. Plants and
animals in freshwater regions are adjusted to the low salt content and would not be able to
survive in areas of high salt concentration (i.e, ocean). There are different types of
freshwater regions: ponds and lakes, streams and rivers, and wetlands. The following
sections describe the characteristics of these three freshwater zones.

Ponds and Lakes

These regions range in size from just a few square meters to thousands of square
kilometers. Scattered throughout the earth, several are remnants from the Pleistocene
glaciation. Many ponds are seasonal, lasting just a couple of months (such as sessile pools)
while lakes may exist for hundreds of years or more. Ponds and lakes may have limited
species diversity since they are often isolated from one another and from other water
sources like rivers and oceans. Lakes and ponds are divided into three different zones
which are usually determined by depth and distance from the shoreline.

The topmost zone near the shore of a lake or pond is the littoral zone. This zone is the
warmest since it is shallow and can absorb more of the Suns heat. It sustains a fairly diverse
community, which can include several species of algae (like diatoms), rooted and floating
aquatic plants, grazing snails, clams, insects, crustaceans, fishes, and amphibians. In the case
of the insects, such as dragonflies and midges, only the egg and larvae stages are found in
this zone. The vegetation and animals living in the littoral zone are food for other creatures
such as turtles, snakes, and ducks.

The near-surface open water surrounded by the littoral zone is the limnetic zone. The
limnetic zone is well-lighted (like the littoral zone) and is dominated by plankton, both

www.iasbaba.com Page 68
IASbabas 60 DAY PLAN- UPSC PRELIMS 2017
ENVIRONMENT (Subject Wise Compilation)

phytoplankton and zooplankton. Plankton are small organisms that play a crucial role in the
food chain. Without aquatic plankton, there would be few living organisms in the world, and
certainly no humans. A variety of freshwater fish also occupy this zone.

Plankton have short life spanswhen they die, they fall into the deep-water part of the
lake/pond, the profundal zone. This zone is much colder and denser than the other two.
Little light penetrates all the way through the limnetic zone into the profundal zone. The
fauna are heterotrophs, meaning that they eat dead organisms and use oxygen for cellular
respiration.

Temperature varies in ponds and lakes seasonally. During the summer, the temperature can
range from 4 C near the bottom to 22 C at the top. During the winter, the temperature at
the bottom can be 4 C while the top is 0 C (ice). In between the two layers, there is a
narrow zone called the thermocline where the temperature of the water changes rapidly.
During the spring and fall seasons, there is a mixing of the top and bottom layers, usually
due to winds, which results in a uniform water temperature of around 4 C. This mixing also
circulates oxygen throughout the lake. Of course there are many lakes and ponds that do
not freeze during the winter, thus the top layer would be a little warmer.

Q.89) Scientists have discovered new species belonging to the genus Nyctibatrachus. It is a
genus of?

a) Lizards endemic to Western Ghats


b) Frogs endemic to Western Ghats
c) Specific Plants endemic to Western Ghats
d) None of the above

Q.89) Solution (b)

Scientists have discovered seven new frog species belonging to the genus Nyctibatrachus,
commonly known as Night Frogs. This find is a result of five years of extensive explorations
in the Western Ghats global biodiversity hotspot in India. Four out of seven of the new
species are miniature-sized frogs (12.2-15.4 mm), which can comfortably sit on a coin or a
thumbnail. These are among the smallest known frogs in the world.

Unlike other frogs in the genus that are predominantly stream dwelling, the new miniature
frogs were found under damp forest leaf litter or marsh vegetation.

www.iasbaba.com Page 69
IASbabas 60 DAY PLAN- UPSC PRELIMS 2017
ENVIRONMENT (Subject Wise Compilation)

Listed as Vulnerable under IUCN because its Extent of Occurrence is less than 20,000 km2,
its distribution is severely fragmented, and there is continuing decline in the extent and
quality of its forest habitat.

http://timesofindia.indiatimes.com/home/environment/flora-fauna/indian-scientists-
discover-four-new-species-of-smallest-known-frogs-from-the-western-
ghats/articleshow/57281095.cms

Q.90) State of Worlds Forests Report is published by

a) World Wildlife Fund (WWF)


b) United Nations Environment Programme (UNEP)
c) Food and Agricultural Organization (FAO)
d) World Bank

Q.90) Solution (c)

State of the Worlds Forests 2016

Forests and agriculture: land-use challenges and opportunities

Forests and trees support sustainable agriculture. They stabilize soils and climate, regulate
water flows, give shade and shelter, and provide a habitat for pollinators and the natural
predators of agricultural pests. They also contribute to the food security of hundreds of
millions of people, for whom they are important sources of food, energy and income. Yet,

www.iasbaba.com Page 70
IASbabas 60 DAY PLAN- UPSC PRELIMS 2017
ENVIRONMENT (Subject Wise Compilation)

agriculture remains the major driver of deforestation globally, and agricultural, forestry and
land policies are often at odds.

State of the Worlds Forests (SOFO) 2016 shows that it is possible to increase agricultural
productivity and food security while halting or even reversing deforestation, highlighting the
successful efforts of Costa Rica, Chile, the Gambia, Georgia, Ghana, Tunisia and Viet Nam.
Integrated land-use planning is the key to balancing land uses, underpinned by the right
policy instruments to promote both sustainable forests and agriculture.

The UN General Assembly proclaimed 2011 as the "International Year of Forests"

The International Day of Forests was established on the 21st day of March, by resolution
of the United Nations General Assembly on November 28, 2012

The World Forestry Congress (WFC) is the largest and most significant gathering of the
world's forestry sector and it has been held every six years since 1926 under the auspices of
the Food and Agriculture Organization of the United Nations (FAO), organized by the
government of the host country. It is a forum for the sharing of knowledge and experience
regarding the conservation, management and use of the world's forests, and covers such
issues as international dialogue, socio-economic and institutional aspects,
and forest policies.

Q.91) Consider the following statements regarding BIOFIN

1. BIOFIN, Biodiversity Finance is a global partnership to help government plan and pay
for its actions on biodiversity conservation and sustainable use.
2. The United Nations Environment Programme (UNEP) is implementing a global multi
country project on Biodiversity Finance Initiative (BIOFIN)
3. India has joined BIOFIN looking at the financing options to reverse biodiversity loss

Which of the given statements is/are correct?

a) 1 and 3
b) 2 and 3
c) 1 and 3
d) 1, 2 and 3

Q.91) Solution (c)

www.iasbaba.com Page 71
IASbabas 60 DAY PLAN- UPSC PRELIMS 2017
ENVIRONMENT (Subject Wise Compilation)

The United Nations Development Programme (UNDP) is implementing a global multi


country project on Biodiversity Finance Initiative (BIOFIN)

http://envfor.nic.in/sites/default/files/press-releases/Press%20Release_BIOFIN.pdf

Q.92) Recently, our scientists of the Botanical Survey of India (BSI) have found a new
species of Zingiber (commonly referred as Ginger) having medicinal properties. In which
part of the country it was discovered?

a) Andaman and Nicobar Islands


b) Eastern Ghats
c) Western Ghats
d) Northern Himalayas

Q.92) Solution (a)

Scientists of the Botanical Survey of India (BSI) have found a new species of Zingiber
(commonly referred as Ginger) from the Andaman and Nicobar Islands.

The species Zingiber pseudosquarrosum, new to science, belonging to genus Zingiber, was
already used by the local Particularly Vulnerable Tribal Groups ( PVTGs) of the Andamans for
its medicinal values.

During an expedition to north and middle Andaman, one of local guides, who was a
Nicobarese, shared his traditional knowledge about this species, which was so far hidden
from science.

After collecting and systematically analysing the species, scientists found that the new
species indeed possesses ethno-medicinal uses.

The fresh extract [juice] of fleshy tuberous roots is used to treat abdominal pain and anti-
helminthic troubles by Nicobarese and certain other tribal communities,.

http://www.thehindu.com/sci-tech/energy-and-environment/New-Ginger-species-with-
medicinal-properties-found-in-Andamans/article17009348.ece

Q.93) In a major breakthrough in eco-conservation, forest officials in Munnar have


spotted two critically endangered impatiens plant species on the peripheries of the
Eravikulam National Park. Consider the statements regarding Impatiens Plant Species:

www.iasbaba.com Page 72
IASbabas 60 DAY PLAN- UPSC PRELIMS 2017
ENVIRONMENT (Subject Wise Compilation)

1. They are also called as jewel weeds


2. All Impatiens taste bitter and seem to be slightly toxic upon ingestion, causing
intestinal ailments like vomiting and diarrhea.
3. Impatiens are non-invasive species

Which of the given statements is/are correct?

a) 1 and 3
b) 2 and 3
c) 1 and 2
d) 1, 2 and 3

Q.93) Solution (c)

Impatiens, also called jewel weeds, are seen in pristine forests where moisture content and
relative humidity are high.

They belong to The Balsaminaceae (commonly known as the Balsam family) are
a family of dicotyledonous plants, comprising two genera: Impatiens, which consists of
1000+ species, and Hydrocera, consisting of 1 species

One of the impatience, Himalayan balsam (Impatiens glandulifera) is an invasive terrestrial


plant species.

All Impatiens taste bitter and seem to be slightly toxic upon ingestion, causing intestinal
ailments like vomiting and diarrhea. The toxic compounds have not been identified but are
probably the same as those responsible for the bitter taste, likely might
be glycosides or alkaloids.

http://www.thehindu.com/todays-paper/tp-national/two-endangered-plant-species-
spotted/article17370078.ece

Q.94) The India-based Neutrino Observatory (INO) Project is a multi-institutional effort


aimed at building a world-class underground laboratory with a rock cover of approx.1200
m for non-accelerator based high energy and nuclear physics research in India. It is located
in

a) The Bodi West Hills region


b) The Anamalai Hills region
c) The Nilgiri Hills region
d) The Kalrayan Hills region

www.iasbaba.com Page 73
IASbabas 60 DAY PLAN- UPSC PRELIMS 2017
ENVIRONMENT (Subject Wise Compilation)

Q.94) Solution (a)

The proposed site for INO is located in the Bodi West Hills region, about 2 km from the
nearest village Pudukottai in Pottipuram Panchayat, Theni District of Tamil Nadu. The
nearest major city is Madurai about 110 km away. It is also the nearest airport and a major
railway station.

Q.95) Consider the following statements

1. Core zone of biosphere consists of an undistributed and legally protected ecosystem


2. Buffer zone of a biosphere reserve is an area of active cooperation between the
management and local people

Which of the statements given above is/are correct?

a) 1 Only
b) 2 Only
c) Both
d) None

Q.95) Solution (a)

Core Zone

Core zone must contain suitable habitat for numerous plant and animal species, including
higher order predators and may contain centres of endemism. Core areas often conserve
the wild relatives of economic species and also represent important genetic reservoirs
having exceptional scientific interest. A core zone being National Park or
Sanctuary/protected/regulated mostly under the Wildlife (Protection) Act, 1972. Whilst
realizing that perturbation is an ingredient of ecosystem functioning, the core zone is to be
kept free from l human pressures external to the system.

Buffer Zone

The buffer zone, adjoins or surrounds core zone, uses and activities are managed in this area
in the ways that help in protection of core zone in its natural condition. These uses and
activities include restoration, demonstration sites for enhancing value addition to the
resources, limited recreation, tourism, fishing, grazing, etc; which are permitted to reduce
its effect on core zone. Research and educational activities are to be encouraged. Human

www.iasbaba.com Page 74
IASbabas 60 DAY PLAN- UPSC PRELIMS 2017
ENVIRONMENT (Subject Wise Compilation)

activities, if natural within BR, are likely to continue if these do not adversely affect the
ecological diversity.

Transition Zone

The transition area is the outermost part of a biosphere reserve. This is usually not delimited
one and is a zone of cooperation where conservation knowledge and management skills are
applied and uses are managed in harmony with the purpose of the biosphere reserve. This
includes settlements, crop lands, managed forests and area for intensive recreation and
other economic uses characteristics of the region.

Q.96) Consider the following regarding Seemai Karuvelam

1. It is an invasive species of plant which is harmful for environment


2. It is native to Indian subcontinent
3. It has one of the deepest roots

Which of the statements given above is/are correct?

a) 1 and 2
b) 2 and 3
c) 1 and 3
d) 1, 2 and 3

Q.96) Solution (c)

In news: The Madurai Bench of the Madras High Court, directed the State government to
bring an Act with Prohibitory and Penal clauses to eradicate seemai karuvelam (Prosopis
Juliflora) trees within two months.

It is native to Mexico, South America and the Caribbean. It has become established as
an invasive weed in Africa, Asia, Australia and elsewhere

Its roots are able to grow to a great depth in search of water: in 1960, they were discovered
at a depth of 53 meters (175 feet) at an open-pit mine near Tucson, Arizona, putting them
among the deepest known roots.

http://www.thehindu.com/news/national/tamil-nadu/a-saviourturnedvillain-whose-
tentacles-spread-far-and-wide/article17379253.ece

www.iasbaba.com Page 75
IASbabas 60 DAY PLAN- UPSC PRELIMS 2017
ENVIRONMENT (Subject Wise Compilation)

Q.97) Consider the following statements

1. Egg laying mammals are known as monotremes


2. The only surviving examples of monotremes are all indigenous to Australia and New
Guinea
3. Monotremes are cold-blooded

Which of the statements given above is/are incorrect?

a) Only 2
b) 1, 2 and 3
c) Only 3
d) None

Q.97) Solution (c)

Mammals are always thought of giving birth to young ones but sometimes it can be
different too. Can you ever think of mammals laying eggs instead of giving birth to babies?
Yes, there are egg laying mammals and they are known as monotremes.

As suggested by egg laying mammal facts, only five species of monotremes are extant, one
of which is duck billed platypus and remaining four belong to the echidna family. These
echidnas are also called as spiny anteaters because they like to have ants and termites in
their diet. Also, due to their pointing spines, they are called spiny anteaters. They are quite
similar with other mammals in few things, but differ in most of the characteristics.
Sometimes, they behave like reptiles, while at other times; they show the characteristics of
birds.

Like other mammals, monotremes are warm-blooded.

The existing monotreme species are the platypus and four species of echidnas (Total 5)

Differences between Platypus and Echidna

Though the echidnas and platypuses share similarities as they both are monotremes, but
there do exist certain differences between them. A female platypus lays eggs in the special
holes or burrows while a female echidna, on the other hand, lays egg in her pouch. Female
platypus can lay two eggs at a time while an echidna can lay only one egg at a time. There
are many other differences in their body structure, feeding habits, diet and so on.

www.iasbaba.com Page 76
IASbabas 60 DAY PLAN- UPSC PRELIMS 2017
ENVIRONMENT (Subject Wise Compilation)

Q.98) Scientists have discovered a new species named Kani maranjandu after the Kani
tribe in Kerala. Identify the species?

a) Lizard
b) Frog
c) Tree-living crab
d) Forest Cat

Q.98) Solution (c)

Scientists have discovered a new species of long legged, tree-dwelling crabs in Western
Ghats of Kerala.

The new species named Kani maranjandu after the Kani tribe in Kerala, are substantially
different from other congeners.

http://www.thehindu.com/news/national/new-tree-living-crab-species-found-in-
kerala/article17809088.ece

Q.99) Consider the following regarding Eco-Sensitive Zones (ESZ)

1. The rules of Eco- Sensitive Zones are declared under Wildlife Protection Act, 1972
2. Flying over protected areas in an aircraft or hot air balloon is prohibited
3. Felling of trees and Rainwater harvesting are regulated around ESZ

Which of the statements given above is/are incorrect?

a) Only 3
b) 1 and 3
c) 1, 2 and 3
d) None

Q.100) Solution (b)

The Ministry of Environment and Forests (MoEF) has come out with new guidelines to
create eco-sensitive zones (ESZs) around Protected Areas to prevent ecological damage
caused due to developmental activities around National Parks and Wildlife Sanctuaries.

www.iasbaba.com Page 77
IASbabas 60 DAY PLAN- UPSC PRELIMS 2017
ENVIRONMENT (Subject Wise Compilation)

The guidelines include a broad list of activities that could be allowed, promoted, regulated
or promoted. This is an important checklist for conservationists to keep in mind while
identifying threats in ESZs.

ESZs are based on the rules of Environment Protection Act, 1986

The guidelines said activities, including commercial mining, setting of saw mills and
industries causing pollution, commercial use of firewood and major hydro-power projects,
are prohibited in such areas.

It also prohibits tourism activities like flying over protected areas in an aircraft or hot air
balloon, and discharge of effluents and solid waste in natural water bodies or terrestrial
areas.

Felling of trees, drastic change in agriculture systems and commercial use of natural water
resources, including groundwater harvesting and setting up of hotels and resorts, are the
activities regulated in the areas.

Activities permitted in the areas include ongoing agriculture and horticulture practices by
local communities, rainwater harvesting, organic farming, adoption of green technology and
use of renewable energy sources.

The width of the ESZ and type of regulation may vary from protected area to area. However,
as a general principle, the width of the ESZ could go up to 10 kms around the protected
area. The ministry said all states and union territories were asked to forward site-specific
proposals to set up ESZs. But only few states have forwarded the proposals. This ministry
after careful consideration, has therefore, decided to frame guidelines to facilitate the
state/union territory for declaration of eco-sensitive zones around national parks and wild
life sanctuaries.

http://www.moef.gov.in/sites/default/files/1%20Guidelines%20for%20Eco-
Sensitive%20Zones%20around%20Protected%20Areas.pdf

Q.101) Identify the correct statement regarding Jerdons Courser

a) It is a nocturnal bird endemic to Western Ghats


b) It is a nocturnal bird endemic to Eastern Ghats
c) It is a nocturnal bird endemic to Andaman and Nicobar
d) It is a nocturnal bird endemic to Northern Himalayas

www.iasbaba.com Page 78
IASbabas 60 DAY PLAN- UPSC PRELIMS 2017
ENVIRONMENT (Subject Wise Compilation)

Q.101) Solution (b)

Jerdon's courser (Rhinoptilus bitorquatus) is a nocturnal bird belonging to


the pratincole and courser family Glareolidae endemic to India. The bird was discovered by
the surgeon-naturalist Thomas C. Jerdon in 1848 but not seen again until its rediscovery in
1986. This courser is a restricted-range endemic found locally in India in the Eastern
Ghats of Andhra Pradesh. It is currently known only from the Sri Lankamalleswara Wildlife
Sanctuary, where it inhabits sparse scrub forest with patches of bare ground.

http://timesofindia.indiatimes.com/city/hyderabad/extinct-birds-to-fly-from-new-
habitat/articleshow/56770355.cms

Q.102) The Montreux Record is a register of wetland sites on the List of Wetlands of
International Importance where changes in ecological character have occurred, are
occurring, or are likely to occur as a result of technological developments, pollution or
other human interference. It is maintained as part of the Ramsar List.

List of Indian wetlands of International importance included in the Montreux Record are:

1. Chilika Lake
2. Loktak Late
3. Keoladeo National Park
4. Wular Lake

Select the correct code

a) 1 and 2
b) 2, 3 and 4
c) 2 and 4
d) 1, 2 and 4

Q.102) Solution (c)

The Montreux Record is a register of wetland sites on the List of Wetlands of International
Importance where changes in ecological character have occurred, are occurring, or are likely
to occur as a result of technological developments, pollution or other human interference. It
is maintained as part of the Ramsar List.

List of Indian wetlands of International importance included in the Montreux Record are

Keoladeo National Park, Rajasthan

www.iasbaba.com Page 79
IASbabas 60 DAY PLAN- UPSC PRELIMS 2017
ENVIRONMENT (Subject Wise Compilation)

Loktak Lake, Manipur.

Note: Chilika Lake was removed from the register in 2002 in light of the improved conditions
of the lake. In fact, Chilika lake is the first Ramsar site in Asia to be removed from the
Montreux record

http://ramsar.rgis.ch/cda/en/ramsar-documents-montreux-montreux-
record/main/ramsar/1-31-118%5E20972_4000_0__

Q.103) Consider the following about The Slim Ali Centre for Ornithology and Natural
History (SACON)

1. It is a statutory body established under Environment Protection Act, 1986


2. It creates data bank on Indian ornithology and natural history,
3. It was established in 1990 due to concerted efforts of Ministry of Environment and
Forests and Bombay Natural History Society

Which of the statements given above is/are correct?

a) 1 and 3
b) 2 and 3
c) 1 and 2
d) 1, 2 and 3

Q.103) Solution (b)

The necessity for the study and conservation of birds in particular, and wildlife and
biodiversity in general, prompted the Ministry of Environment and Forests, Government of
India to establish the Salim Ali Centre for Ornithology & Natural History as a public - NGO
partnership between the MoEF, and the Bombay Natural History Society (BNHS).

The Salim Ali Centre for Ornithology & Natural History (SACON) is a Society registered
(autonomous body) in 1990 under the Societies registration Act, with the object of
establishing and developing a Centre of Excellence to assist, institute, conduct and promote
scientific research in ornithology, and of species, habitats and ecosystems with and within
which avifauna coexist, and developing scientific solutions to species, habitat and landscape
conservation problems that are sensitive to the socio-economic realities and aspirations of
the people.

The core objectives of SACON, to which all work of the organisation are directed, are:

www.iasbaba.com Page 80
IASbabas 60 DAY PLAN- UPSC PRELIMS 2017
ENVIRONMENT (Subject Wise Compilation)

Design and conduct research in ornithology covering all aspects of biodiversity and
natural History,
Develop and conduct regular courses in ornithology and natural history
for MSc, MPhil and PhD candidates and short-term orientation courses in the above
subjects,
Create a data bank on Indian ornithology and natural history,
Disseminate knowledge relating to ornithology and natural history for the benefit of
the community.
To confer honorary awards and other distinctions to persons who have rendered
outstanding services in the fields of ornithology and natural history.

Q.104) Mangroves for the Future (MFF) is a unique partner-led initiative to promote
investment in coastal ecosystem conservation for sustainable development. It is
developed by

a) IUCN and UNDP


b) IUCN and WWF
c) UNESCO and IUCN
d) UNEP and IUCN

Q.104) Solution (a)

The devastation caused by the Indian Ocean tsunami of December 2004 laid bare the vital
link between coastal ecosystems and human livelihoods. It was United States President Bill
Clintons vision that rebuilding in tsunami-hit areas should improve natural infrastructure
and strengthen resilience against future natural disasters.

In response to this vision, IUCN (International Union for Conservation of Nature) and the
United Nations Development Programme (UNDP) developed Mangroves for the Future in
2006.

Since then, MFF has grown to include eight institutional partners, plus a growing number of
countries. At the launch of the Tsunami Legacy report* at the United Nations in New York in
April 2009, Bill Clinton acclaimed MFF as one of the most positive and forward-looking
developments of the post-tsunami period.

Mangroves for the Future (MFF) is a unique partner-led initiative to promote investment
in coastal ecosystem conservation for sustainable development. Co-chaired by IUCN and
UNDP, MFF provides a platform for collaboration among the many different agencies,

www.iasbaba.com Page 81
IASbabas 60 DAY PLAN- UPSC PRELIMS 2017
ENVIRONMENT (Subject Wise Compilation)

sectors and countries which are addressing challenges to coastal ecosystem and livelihood
issues. The goal is to promote an integrated ocean-wide approach to coastal management
and to building the resilience of ecosystem-dependent coastal communities.

Mangroves are the flagship of the initiative, but MFF is inclusive of all types of coastal
ecosystem, such as coral reefs, estuaries, lagoons, sandy beaches, seagrass and wetlands.

Member countries: Bangladesh, Cambodia, India, Indonesia, Maldives, Myanmar, Pakistan,


Seychelles, Sri Lanka, Thailand, and Viet Nam

Q.105) Consider the following

1. Grassland
2. Wetland
3. Mangroves
4. Estuary
5. Riparian Zone

Which of the above are Ecotone Regions or Transitional Zones?

a) 2, 3 and 4
b) 1, 2, 3 and 4
c) 1, 2, 3, 4 and 5
d) 2, 4 and 5

Q.105) Solution (c)

An ecotone is a zone of junction or a transition area between two biomes [diverse


ecosystems]. It is where two communities meet and integrate.

For e.g. the mangrove forests represent an ecotone between marine and terrestrial
ecosystem. Other examples are grassland (between forest and desert), estuary (between
fresh water and salt water) and river bank or marsh land (between dry and wet).

An ecotone may appear on the ground as a gradual blending of the two communities across
a broad area, or it may manifest itself as a sharp boundary line.

A riparian zone or riparian area is the interface between land and a river or stream.

www.iasbaba.com Page 82
IASbabas 60 DAY PLAN- UPSC PRELIMS 2017
ENVIRONMENT (Subject Wise Compilation)

Q.106) You are trying to protect a National Park/Sanctuary from various pressures
including a dam proposal and widening of a highway. Consider the following regarding
this:

1. You can directly approach National Green Tribunal in regard to your case
2. You must have a lawyer to represent you for this case in front of National Green
Tribunal

Which of the above is/are correct w.r.t rules and procedure to approach National Green
Tribunal?

a) 1 Only
b) 2 Only
c) Both
d) None

Q.106) Solution (d)

Unfortunately both are wrong :P Why?

Because NGT is not empowered to hear matters pertaining to issues coming under the
ambit of the Wildlife (Protection) Act, 1972, which is applicable in case of National Parks,
Sanctuaries and Tiger Reserves. It would be appropriate to approach either the High Court in
your State or the Supreme Court. Please consult a competent lawyer for advice.

The NGT has the power to hear all civil cases relating to environmental issues and questions
that are linked to the implementation of laws listed in Schedule I of the NGT Act. These
include the following:

The Water (Prevention and Control of Pollution) Act, 1974;


The Water (Prevention and Control of Pollution) Cess Act, 1977;
The Forest (Conservation) Act, 1980;
The Air (Prevention and Control of Pollution) Act, 1981;
The Environment (Protection) Act, 1986;
The Public Liability Insurance Act, 1991;
The Biological Diversity Act, 2002.

This means that any violations pertaining only to these laws, or any order / decision taken
by the Government under these laws can be challenged before the NGT. Importantly, the
NGT has not been vested with powers to hear any matter relating to the Wildlife
(Protection) Act, 1972, the Indian Forest Act, 1927 and various laws enacted by States
relating to forests, tree preservation etc. Therefore, specific and substantial issues related to
these laws cannot be raised before the NGT. You will have to approach the State High Court

www.iasbaba.com Page 83
IASbabas 60 DAY PLAN- UPSC PRELIMS 2017
ENVIRONMENT (Subject Wise Compilation)

or the Supreme Court through a Writ Petition (PIL) or file an Original Suit before an
appropriate Civil Judge of the taluk where the project that you intend to challenge is
located.

Q.107) Which of the following correctly defines Bio-mining?

a) Use of genetic information of ores to extract minerals


b) Extraction of minerals using enzymes
c) Use of microorganisms in the recovery of minerals from ores
d) Use of biomedical devices in the recovery of minerals from ores

Q.107) Solution (c)

Biomining is an approach to the extraction of desired minerals from ores.

Microorganisms are used to leach out the minerals, rather than the traditional methods of
extreme heat or toxic chemicals, which have a deleterious effect on the environment.

In news: http://www.thehindu.com/news/cities/Tiruchirapalli/Bio-mining-helps-in-tackling-
garbage-problem/article16438953.ece

Q.108) Certain special conditions which are necessary prerequisites for the growth and
development of corals. Consider the following in this regard:

1. For the growth and development of corals the surface temperature of the ocean
must be below 20C
2. Shallow continental shelf
3. Corals need saltwater to survive
4. Low sediments
5. If the salinity of the ocean water is very high, the lime content is bound to be low,
which is favourable for the growth of corals

Select the correct code

a) 1, 2, 3, 4 and 5
b) 1, 3 and 4 only
c) 2, 3 and 4 only
d) 2, 3 and 5 only

www.iasbaba.com Page 84
IASbabas 60 DAY PLAN- UPSC PRELIMS 2017
ENVIRONMENT (Subject Wise Compilation)

Q.108) Solution (c)

There are certain special conditions which are necessary prerequisites for the growth and
development of corals. That is why the reef building corals and their associates are not
uniformly deposited throughout the tropical warm ocean waters.

For the growth of corals in such large numbers, as is necessary to give rise to coral deposits,
there must be a reasonable balance of favourable conditions. The most important of these
conditions are the following:

1. For the growth and development of corals the surface temperature of the ocean must be
above 20C. Corals cannot live if the temperature of the sea water falls below this.

2. The water must be shallow, with a depth not exceeding 25-30 fathoms. As we know, with
increasing depth, the amount of calcium as well as the temperature of water goes on
decreasing, so that the coral polyps and other sea creatures which live on calcium carbonate
do not thrive. That is why the reef building corals live in shallow waters and on the surface
of the seas.

3. The water must be normally saline, and, therefore, along the ocean margins where the
water becomes fresh by the inflow of rivers, reef building corals do not grow and develop.

4. Corals need clear water that is free from abundant sediments. That is why coral reefs
cannot develop where rivers enter the sea or where wave-erosion causes muddy coastal
water.

5. There must be adequate food supply to nourish the abundant life of the coral reef. The
most favourable condition for this purpose is the presence of continuously flowing ocean
currents which provide to the stationary reef building organisms the much needed food
supply.

For example, extensive coral reefs are found on the east coasts of Australia, Central
America, and Africa, which are washed by warm ocean currents flowing along them. On the
contrary, corals are found only in scattered patches on the west coasts of these continents.

6. If the salinity of the ocean water is very high, the lime content is bound to be low, which
is not favourable for the growth of corals. Therefore the average salinity ranging from 27%
to 40% is ideal for the proper growth of corals and other reef forming creatures.

7. The most essential prerequisite for the formation of coral reefs is the presence of sub-
marine platforms which must lie near the sea shore, or should be attached to some islands.

www.iasbaba.com Page 85
IASbabas 60 DAY PLAN- UPSC PRELIMS 2017
ENVIRONMENT (Subject Wise Compilation)

The depth of water on such platforms should not exceed 50 fathoms. Remember that the
corals build their permanent colonies only on these platforms.

8. Another point to remember is that corals cannot live for long out of water, and are
therefore, rarely found above the low-tide level. On the other hand, their growth is retarded
at depths much exceeding 25 or 30 fathoms.

9. Corals need clear oxygenated water with sufficient supplies of microscopic life as food.

10. Since food supplies are plentiful on the seaside of a growing reef, the corals tend to
grow rather at a fast rate outwards.

As the reefs grow in size, waves wash much broken corals in the form of boulders and sand
over the crest, thus building up a reef-flat, often with sand dunes, upon which vegetation
may grow. Coconut palms add special beauty to these 'Coral Islands'.

It should be borne in mind that "the reefs and atolls are easily 'drowned' if their upward
growth cannot keep place with any submergence (eustatic or tectonic) that may be in
progress."

As regards the rate of growth of corals, it depends on the species, the range being 6 to 45
mm per year. However, according to Arthur Holmes, the average rate of reef growth is
about 14 mm a year.

Q.109) With reference to food chains in ecosystems, consider the following statements:

1. A food chain illustrates the order in which a chain of organisms feed upon each
other.
2. Food chains are found within the populations of a species.
3. A food chain illustrates the numbers of each organism which are eaten by others.

Which of the statements given above is / are correct?

a) 1 only
b) 1 and 2 only
c) 1, 2 and 3
d) None

Q.109) Solution (a)

www.iasbaba.com Page 86
IASbabas 60 DAY PLAN- UPSC PRELIMS 2017
ENVIRONMENT (Subject Wise Compilation)

A food chain illustrates the order in which a chain of organisms feed upon each other. This
statement is correct.

Food chains are found within the populations of a species. (Not correct- Within the
population-Wrong)

A food chain illustrates the numbers of each organism which are eaten by others (it is
wrong) Note: Food web illustrates the number not the food chain)

Q.110) The ecological footprint is a measure of human demand on the Earth's ecosystems.
It measures-

1. How much of the biological capacity of the planet is demanded by a given human
activity or population
2. Ecological Footprints is calculated for overall activity of a nation or population and
activities such as industrialization etc.
3. The disadvantage of Ecological Footprint is that it cannot be calculated for individual
people or area.

Select the incorrect statement/s

a) 1 and 2
b) Only 2
c) 1 and 3
d) Only 3

Q.110) Solution (d)

The Ecological Footprint is a resource accounting tool that measures how much biologically
productive land and sea is used by a given population or activity, and compares this to how
much land and sea is available. Productive land and sea areas support human demands for
food, fibre, timber, energy, and space for infrastructure. These areas also absorb the waste
products from the human economy. The Ecological Footprint measures the sum of these
areas, wherever they physically occur on the planet. The Ecological Footprint is used widely
as a management and communication tool by governments, businesses, educational
institutions, and non-governmental organizations.

Ecological Footprint accounts answer a specific research question: how much of the
biological capacity of the planet is demanded by a given human activity or population? To
answer this question, the Ecological Footprint measures the amount of biologically

www.iasbaba.com Page 87
IASbabas 60 DAY PLAN- UPSC PRELIMS 2017
ENVIRONMENT (Subject Wise Compilation)

productive land and water area an individual, a city, a country, a region, or all of humanity
uses to produce the resources it consumes and to absorb the waste it generates with
todays technology and resource management practices. This demand on the biosphere can
be compared to biocapacity, a measure of the amount of biologically productive land and
water available for human use. Biologically productive land includes areas such as cropland,
forest, and fishing grounds, and excludes deserts, glaciers, and the open ocean

Ecological Footprints can be calculated for individual people, groups of people (such as a
nation), and activities (such as manufacturing a product).

The Ecological Footprint of a person is calculated by considering all of the biological


materials consumed, and all of the biological wastes generated, by that person in a given
year. These materials and wastes each demand ecologically productive areas, such as
cropland to grow potatoes, or forest to sequester fossil carbon dioxide emissions. All of
these materials and wastes are then individually translated into an equivalent number of
global hectares.

Q.111) Consider the statements regarding Coral bleaching

1. It is the loss of zooxanthellae through either expulsion or loss of algal pigmentation.


2. It happens due to increase in ocean temperature only
3. All the corals undergoing bleaching are dead corals

Select the correct code

a) 1 and 3
b) Only 2
c) Only 1
d) 2 and 3

Q.111) Solution (c)

First statement is true. Coral bleaching not only happens because of warm temperature but
can also happen due to Cold temperature. Not all bleaching events are due to warm water.
In January 2010, cold water temperatures in the Florida Keys caused a coral bleaching event
that resulted in some coral death. Water temperatures dropped 12.06 degrees
Fahrenheit lower than the typical temperatures observed at this time of year. Researchers
will evaluate if this cold-stress event will make corals more susceptible to disease in the
same way that warmer waters impact corals.

www.iasbaba.com Page 88
IASbabas 60 DAY PLAN- UPSC PRELIMS 2017
ENVIRONMENT (Subject Wise Compilation)

When a coral bleaches, it is not dead. Corals can survive a bleaching event, but they are
under more stress and are subject to mortality.

http://www.theaustralian.com.au/news/nation/great-barrier-reef-scientists-exaggerated-
coral-bleaching/news-story/99810c83f5a420727b12ab255256774b

Q.112) Consider the following

1. Ocean
2. Grasslands
3. Lakes
4. Mangroves

Which one of the following is the correct sequence of ecosystems in the order of
increasing productivity?

a) Oceans, lakes, grasslands, mangroves


b) Mangroves, oceans, grasslands, lakes
c) Mangroves, grasslands, lakes, oceans
d) Oceans, mangroves, lakes, grasslands

Q.112) Solution (a)

Mangroves are high productivity ecosystem. And since it is asked for increasing order, only
option (a) has mangrove in the last.

Q.113) Which of the floral groups are found in India?

1. Lichens
2. Bryophytes
3. Pteridophytes
4. Gymnosperms
5. Angiosperms

Select the correct answers

a) 1, 4 and 5 only
b) 1, 2, 3, 4 and 5

www.iasbaba.com Page 89
IASbabas 60 DAY PLAN- UPSC PRELIMS 2017
ENVIRONMENT (Subject Wise Compilation)

c) 1, 2 and 4 Only
d) 1, 4 and 5 Only

Q.113) Solution (b)

All the floral groups found in India. Simple examples can be read from class 9 NCERT

Q.114) The migration pattern Amur Falcon, known to be one of the longest distances
undertaken by migratory birds has been one of the interesting fields of research among
scientists and ornithologists. Identify the regions where Amur Falcon travels during its
migration?

1. South Africa
2. China
3. Siberia
4. Rajasthan
5. Nagaland

Select the correct code

a) 1, 2 and 5
b) 2, 3, 4 and 5
c) 2, 3 and 5
d) 1, 2, 3, 4 and 5

Q.114) Solution (d)

The bird breeds in south-eastern Siberia and northern China before migrating in large flocks
across India and over the Arabian Sea to winter in southern Africa.

It has been spotted in Rajasthan recently.

The birds that usually travelled through Maharashtra and Gujarat, were for the first time
spotted in Rajasthan in early May

The entire migration route of Amur falcons from Nagaland to South Africa, Mongolia and
back to Nagaland has been scientifically plotted. Scientists now have confirmed that a
satellite-tagged Amur falcon 'Naga' has travelled through Rajasthan.

www.iasbaba.com Page 90
IASbabas 60 DAY PLAN- UPSC PRELIMS 2017
ENVIRONMENT (Subject Wise Compilation)

The Amur falcon a marathon flier whose migrations stretch from the icy environs of
Siberia to the outcrops of South Africa has been spotted in rare areas across the State
(Spotted in Ballari, Kodagu, and the urban sprawls of Mysuru)

Q.115) Bio-magnification refers to increase in concentration of a pollutant from one link in


a food chain to another. Consider the conditions in order for bio-magnification to occur

1. The pollutant must be short-lived


2. The pollutant should be biologically active
3. The pollutant must not be mobile
4. The pollutant should have higher solubility in water

Which of the above are incorrect?

a) 1, 3 and 4
b) 1 Only
c) 1, 2, 3 and 4
d) None of the above

Q.116) Solution (a)

We are concerned about these phenomena because together they mean that even small
concentrations of chemicals in the environment can find their way into organisms in high
enough dosages to cause problems. In order for biomagnification to occur, the pollutant
must be:

long-lived
mobile
soluble in fats
biologically active

If a pollutant is short-lived, it will be broken down before it can become dangerous. If it is


not mobile, it will stay in one place and is unlikely to be taken up by organisms. If the
pollutant is soluble in water it will be excreted by the organism. Pollutants that dissolve in
fats, however, may be retained for a long time. It is traditional to measure the amount of
pollutants in fatty tissues of organisms such as fish. In mammals, we often test
the milk produced by females, since the milk has a lot of fat in it and because the very young
are often more susceptible to damage from toxins (poisons). If a pollutant is not active
biologically, it may biomagnify, but we really don't worry about it much, since it probably
won't cause any problems.

www.iasbaba.com Page 91
IASbabas 60 DAY PLAN- UPSC PRELIMS 2017
ENVIRONMENT (Subject Wise Compilation)

Q.117) As per the notification of Ministry of Environment and Forests, the coastal land up
to 500m from the High Tide Line (HTL) and a stage of 100m along banks of creeks,
estuaries, backwater and rivers subject to tidal fluctuations, is called the Coastal
Regulation Zone(CRZ). Consider the following regarding CRZ

1. The area up to 200 metres from the High Tide Line is earmarked as No Development
Zone
2. Fishing and allied activities are permitted in CRZ-IV

Which of the above is/are correct?

a) 1 Only
b) 2 Only
c) Both
d) None

Q.117) Solution (c)

CRZ-1: these are ecologically sensitive areas these are essential in maintaining ecosystem of
the coast. They lie between low and high tide line. Exploration of natural gas and extraction
of salt are permitted

CRZ-2: these areas form up to the shore line of the coast. Unauthorised structures are not
allowed to construct in this zone.

CRZ-3: rural and urban localities which fall outside the 1 and 2. Only certain activities related
to agriculture even some public facilities are allowed in this zone

CRZ-4: this lies in aquatic area up to territorial limits. Fishing and allied activities are
permitted in this zone. Solid waste should be let off in this zone.

http://envfor.nic.in/legis/crz/crznew.html

Q.118) Which of the following are the methods of ex-situ conservation?

1. Biodiversity Hotspots
2. Community Reserves
3. Zoological Parks
4. Gene Sanctuary

www.iasbaba.com Page 92
IASbabas 60 DAY PLAN- UPSC PRELIMS 2017
ENVIRONMENT (Subject Wise Compilation)

5. Botanical Gardens
6. Sacred Groves
7. Seed banks

Select the correct code

a) 2, 4, 5 and 6 only
b) 2, 3, 4 and 7 only
c) 1, 2, 4 and 7 only
d) 3, 5 and 7 only

Q.118) Solution (d)

In Situ Conservation Methods

In-situ conservation, the conservation of species in their natural habitats, is considered the
most appropriate way of conserving biodiversity.

Conserving the areas where populations of species exist naturally is an underlying condition
for the conservation of biodiversity. That's why protected areas form a central element of
any national strategy to conserve biodiversity.

Methods

Biosphere reserves
National parks
Wild sanctuaries
Biodiversity Hotspots
Gene sanctuary
Community reserves
Sacred groves

Ex Situ Conservation Methods

Ex-situ conservation is the preservation of components of biological diversity outside their


natural habitats. This involves conservation of genetic resources, as well as wild and
cultivated or species, and draws on a diverse body of techniques and facilities. Some of
these include:

Gene banks, e.g. seed banks, sperm and ova banks, field banks;
In vitro plant tissue and microbial culture collections;

www.iasbaba.com Page 93
IASbabas 60 DAY PLAN- UPSC PRELIMS 2017
ENVIRONMENT (Subject Wise Compilation)

Captive breeding of animals and artificial propagation of plants, with possible


reintroduction into the wild; and
Collecting living organisms for zoos, aquaria, and botanic gardens for research and
public awareness.

Q.119) Which among the following can effectively be examples of Secondary Succession?

1. Formation of a new Island after volcanic eruption


2. The renewal of a forest after a fire
3. A flooded land
4. The renewal of a crop after harvesting

Select the correct option

a) 1, 2 and 3
b) 2, 3 and 4
c) 1, 3 and 4
d) All

Q.119) Solution (b)

Examples of secondary succession include:


The renewal of a forest after a fire: The fire itself destroys a majority of different
types of trees and plant life. Because seeds and roots and other plant and tree parts
remain in and on the soil, gradually the plants and trees begin to grow again and
eventually return to the state of the original ecosystem.
The renewal of a crop after harvesting: A crop is completed harvested when it
becomes ripe. Without new seeds being planted, the crop can regenerate the
following year due to the plants and seeds that remained after harvesting.
A forest renews after logging: A large amount of trees were chopped down by
loggers in order to create building materials. Over time, trees grow in and the area
returns to its previous state.
Renewal after disease: A plant population can be very negatively affected by a
variety of infectious plant diseases. If the entire population dies, but the soil and
roots remain, it is possible for secondary succession to occur and for the population
of those plants to return.

www.iasbaba.com Page 94
IASbabas 60 DAY PLAN- UPSC PRELIMS 2017
ENVIRONMENT (Subject Wise Compilation)

A flood can ruin farmlands. However, because the soil remains after the waters
recede, over the course of many years a natural secondary succession can occur and
the vegetation that had previously grown there can grow again.
Plants can be very susceptible to attack from pests, particularly if there is an
overpopulation of those pests. When this occurs, the plant population in one area
can be completely destroyed. However, when the pest overpopulation is resolved,
the plants are able to live again and thrive in the soil in which they previously had
lived.

Q.120) Which of the following adds/add nitrogen to the soil?

1. Excretion of urea by animals


2. Burning of coal by man
3. Death of vegetation

Select the correct answer using the codes given below

a) 1 only
b) 2 and 3 only
c) 1 and 3 only
d) 1, 2 and 3

Q.120) Solution (c)

Animal Waste like Urea, Uric acid and Death of vegetation add nitrogen in the form of
nitrates directly into soil.

Coal combustion adds nitrogen to atmosphere not to soil directly and from there it falls
back to earth in the form of acid rain and acid rain adds nitrogen to soil.

Q.121) Consider the following statements regarding Living Planet Index Report

1. It is published by IUCN annually.


2. The report is compiled with data from the Zoological Society of India (ZSI) to
measure the abundance of biodiversity.

Which of the given statements is/are correct?

a) 1 Only
b) 2 Only

www.iasbaba.com Page 95
IASbabas 60 DAY PLAN- UPSC PRELIMS 2017
ENVIRONMENT (Subject Wise Compilation)

c) Both
d) None

Q.121) Solution (d)

The Living Planet Report is published every two years by the World Wide Fund for
Nature since 1998. It is based on the Living Planet Index and ecological
footprint calculations.

The Living Planet Report is the world's leading, science-based analysis on the health of our
only planet and the impact of human activity. Humanity's demands exceed the Earth's
capacity to sustain us. The latest edition of the Living Planet Report was released in October
2016

Populations of vertebrate animalssuch as mammals, birds, and fishhave declined by


58% between 1970 and 2012. And were seeing the largest drop in freshwater species: on
average, theres been a whopping 81% decline in that time period.

The Living Planet Database (LPD) currently holds time-series data for over 18,000
populations of more than 3,600 mammal, bird, fish, reptile and amphibian species from
around the world, which are gathered from a variety of sources such as journals, online
databases and government reports.

Using a method developed by Zoological Society of London and WWF, these species
population trends are aggregated to produce indices of the state of biodiversity for
communication and informing policy.

Q.122) Consider the following statements about Rio Declaration on Environment and
Development

1. The Rio Declaration proclaims 30 principles.


2. It includes formulations of the precautionary principle and the polluter pays
principle
3. It also includes Environment Impact Assessment

Select the correct answer using below code

a) 1 and 2
b) 2 and 3
c) 1 and 3
d) 1, 2 and 3

www.iasbaba.com Page 96
IASbabas 60 DAY PLAN- UPSC PRELIMS 2017
ENVIRONMENT (Subject Wise Compilation)

Q.122) Solution (b)

The Rio Declaration proclaims 27 principles. It includes formulations of the precautionary


principle (principle 15) and of the polluter pays principle (principle 16).

Rio Declaration on Environment and Development consisted of 27 principles intended to


guide future sustainable development around the world.

1. The role of humans.


2. State sovereignty
3. The Right to development
4. Environmental Protection in the Development Process
5. Eradication of Poverty
6. Priority for the Least Developed
7. State Cooperation to Protect Ecosystem
8. Reduction of Unsustainable Patterns of Production and Consumption
9. Capacity Building for Sustainable Development
10. Public participation
11. National Environmental Legislation
12. Supportive and Open International Economic System
13. Compensation for Victims of Pollution and other Environmental Damage
14. State Cooperation to Prevent environmental dumping
15. Precautionary principle
16. Internalization of Environmental Costs
17. Environmental Impact Assessments
18. Notification of Natural Disaster
19. Prior and Timely Notification
20. Women have a Vital Role
21. Youth Mobilization
22. Indigenous Peoples have a Vital Role
23. People under Oppression
24. Warfare Peace,
25. Development and Environmental Protection
26. Resolution of Environmental Disputes
27. Cooperation between State and People

Some Scholars have regarded the Rio Declaration as Third Generation Human Rights.
http://www.un.org/documents/ga/conf151/aconf15126-1annex1.htm

www.iasbaba.com Page 97
IASbabas 60 DAY PLAN- UPSC PRELIMS 2017
ENVIRONMENT (Subject Wise Compilation)

Q.123) Consider the following

1. Marine Park
2. National Park
3. Coral reefs
4. Mangroves
5. Sand Dunes

Among the following, entities that fall under Coastal Regulation Zone-I (CRZ)

a) 1,2 and 4
b) 2, 3 and 4
c) 1, 3 and 5
d) 1,2,3, 4 and 5

Q.123) Solution (d)

The CRZ Notification, 2011 clearly lists out the areas that fall within the category
of CRZ-I. It includes:-

(i)Ecologically sensitive areas and the geomorphologic features that play a Primary role in
maintaining the integrity of the coast.

Mangroves, in case mangrove area is more than 1000 square metres, a buffer area of
50 metres shall be provided;
Corals and coral reefs and associated biodiversity;
Sand Dunes;
Mudflats which are biologically active;
National parks, marine parks, sanctuaries, reserve forests, wildlife habitats and other
protected areas under the provisions of Wild Life (Protection) Act, 1972 (53 of 1972),
the Forest (Conservation) Act, 1980 (69 of 1980)or Environment (Protection) Act,
1986 (29 of 1986); including Biosphere Reserves encompassing;

o Salt Marshes;
o Turtle nesting grounds;
o Horse shoe crabs habitats;
o Sea grass beds;
o Nesting grounds of birds;
o Areas or structures of archaeological importance and heritage sites;

www.iasbaba.com Page 98
IASbabas 60 DAY PLAN- UPSC PRELIMS 2017
ENVIRONMENT (Subject Wise Compilation)

(ii) The area between Low Tide Line and High Tide Line.

Q.124) India is party to the following international agreements which have bearing on
biodiversity?

1. FAO International Treaty on Plant Genetic Resources for food and agriculture
2. World Heritage Convention
3. World Trade Organization
4. UN Convention to Combating Desertification
5. UN Law of the Seas

Select the correct code

a) 2 and 4
b) 1, 2 and 4
c) 2, 3, 4 and 5
d) 1, 2, 3, 4 and 5

Q.124) Solution (d)

Article 6 of the treaty on Sustainable Use of Plant Genetic Resources has this clause
pursuing fair agricultural policies that promote use of agricultural biological diversity

UNCCD collaborates closely with Convention on Biological Diversity (CBD). So, UNCCD has a
bearing on Biodiversity

Seven international conventions focus on biodiversity issues: the Convention on Biological


Diversity (year of entry into force: 1993), the Convention on Conservation of Migratory
Species, the Convention on International Trade in Endangered Species of Wild Fauna and
Flora (1975), the International Treaty on Plant Genetic Resources for Food and Agriculture
(2004), the Ramsar Convention on Wetlands (1971), the World Heritage Convention (1972)
and the International Plant Protection Convention (1952).

WTO because of TRIPS rules.

The Convention on Biological Diversity complements UNCLOS in explicitly providing for the
conservation of biological diversity, the sustainable use of its components and the fair and
equitable sharing of benefits arising out of the use of genetic resources.

www.iasbaba.com Page 99
IASbabas 60 DAY PLAN- UPSC PRELIMS 2017
ENVIRONMENT (Subject Wise Compilation)

Q.125) Consider the following

1. Food Processing
2. Printing Press
3. Pulp and Paper
4. Sugar
5. Healthcare Establishments

Which of the given industries come under RED CATEGORY INDUSTRY?

a) 1, 2 and 4
b) 2, 3, 4 and 5
c) 3, 4 and 5
d) 1, 2, 3, 4 and 5

Q.125) Solution (c)

The Ministry of Environment, Forest and Climate Change (MoEFCC) has developed the
criteria of categorization of industrial sectors based on the Pollution Index which is a
function of the emissions (air pollutants), effluents (water pollutants), hazardous wastes
generated and consumption of resources.

Industrial Sectors having Pollution Index score of 60 and above - Red category
Industrial Sectors having Pollution Index score of 41 to 59 Orange category
Industrial Sectors having Pollution Index score of 21 to 40 Green category
Industrial Sectors having Pollution Index score incl.&upto 20 - White category

Refer here for all the categories- http://pib.nic.in/newsite/PrintRelease.aspx?relid=137373

Q.126) Green Digital Finance Alliance is a unique partnership between

a) IUCN and Ant Financial Services Group


b) World Bank and Ant Financial Services Group
c) UNEP and Ant Financial Services Group
d) UNDP and Ant Financial Services Group

Q.126) Solution (c)

www.iasbaba.com Page 100


IASbabas 60 DAY PLAN- UPSC PRELIMS 2017
ENVIRONMENT (Subject Wise Compilation)

The Green Digital Finance Alliance, a ground-breaking initiative to harness digital


technologies in catalysing financing that addresses global environmental challenges, was
launched today at the World Economic Forum Annual Meeting in Davos, Switzerland by its
two founders, Ant Financial Services Group (Ant Financial), Chinas leading online and
mobile financial services provider, and UN Environment.

Q.127) Scientists have discovered two new species Sholicola and Montecincla in the
Western Ghats. They are-

a) New species of Lizard


b) New species of Song Bird
c) New species of Frog
d) New species of Bat

Q.127) Solution (b)

http://www.thehindu.com/sci-tech/science/Music-to-the-ears-New-species-of-songbird-
found/article17093802.ece

Q.128) Which of the following protected areas are formed as per the guidelines of The
Wildlife (Protection) Act, 1972?

1. National Park
2. Biosphere Reserve
3. Wildlife Sanctuary
4. Conservation Reserves
5. Community Reserves

Select the correct code

a) 1, 2, 3 and 4
b) 1, 3, 4 and 5
c) 1, 2, 3 and 5
d) All

Q.128) Solution (b)

www.iasbaba.com Page 101


IASbabas 60 DAY PLAN- UPSC PRELIMS 2017
ENVIRONMENT (Subject Wise Compilation)

Biosphere Reserves are areas of terrestrial and coastal ecosystems which are internationally
recognized within the framework of the Man and the Biosphere (MAB) programme of the
UNESCO and are not formed according to the guidelines of the Wildlife (protection) Act,
1972 and may have one more national parks or wildlife sanctuaries in it. Under the MAB
programme there is a World Network of Biosphere Reserves (WNBR) and within this
network, exchange of information, experience and personnel is allowed.

Q.129) Consider the following statements with respect to Green Bonds

1. Green bond is a debt instrument issued by an entity for raising funds from investors
2. The funds generated are used to fund both green and non-green projects
3. A green bond is a tax-exempt bond issued by federally qualified organizations for the
development of brownfield sites.

Select the correct answer using the code given below

a) 1 and 3 only
b) 2 and 3 only
c) All the above
d) None of the above

Q.129) Solution (a)

Fund generated from bonds are used only to fund green projects.
A green bond is a tax-exempt bond issued by federally qualified organizations or by
municipalities for the development of brownfield sites. Brownfield sites are areas of land
that are underutilized, have abandoned buildings or are underdeveloped, often containing
low levels of industrial pollution. Green bonds are short for qualified green building and
sustainable design project bonds.

Green bonds are created to encourage sustainability and the development of brownfield
sites. More specifically, green bonds finance projects aimed at energy efficiency, pollution
prevention, sustainable agriculture, fishery and forestry, the protection of aquatic and
terrestrial ecosystems, clean transportation, sustainable water management, and the
cultivation of environmentally friendly technologies.

The tax-exempt status makes purchasing a green bond a more attractive investment
compared to a comparable taxable bond, providing a monetary incentive to tackle

www.iasbaba.com Page 102


IASbabas 60 DAY PLAN- UPSC PRELIMS 2017
ENVIRONMENT (Subject Wise Compilation)

prominent social issues such as climate change and a movement to renewable sources of
energy.

Q.130) Which of the following is/are the man-made Ramsar sites of India?

1. Ropar wetland
2. Harike reservoir
3. Bhoj Wetland
4. Ashtamudi

Select the correct answer using the code given below.

a) 2 only
b) 2 and 3
c) 1, 2 and 3
d) All of the above

Q.130) Solution (c)

The list of Ramsar sites (related to wetland) in India comprises Indian wetlands deemed to
be of "international importance" under the Ramsar Convention. For a full list of all Ramsar
sites worldwide, see the Ramsar list of wetlands of international importance.

According to WWF-India, wetlands are one of the most threatened of all ecosystems in
India. Loss of vegetation, salinization, excessive inundation, water pollution, invasive
species, excessive development and road building, have all damaged the countrys wetlands.

Ashtamudi is a natural backwater in Kollam district in Kerala. Rest are manmade reservoirs.

The Bhoj Wetland consists of two lakes located in the city of Bhopal, the capital of the
central Indian state of Madhya Pradesh. The two lakes are the Bhojtal and the Lower Lake,
which lie to the west of the city center. It is a manmade reservoir. A total of more than
20,000 birds are observed annually. Bhoj Wetland was recognized as a wetland of
international importance under the Ramsar Convention of 1971 in 2002.

Ropar wetland- A human-made wetland of lake and river formed by the 1952 construction
of a barrage for diversion of water from the Sutlej River for drinking and irrigation supplies.
The site is an important breeding place for the nationally protected Smooth Indian Otter,
Hog Deer, Sambar, and several reptiles, and the endangered Indian Pangolin (Manis
crassicaudata) is thought to be present.

www.iasbaba.com Page 103


IASbabas 60 DAY PLAN- UPSC PRELIMS 2017
ENVIRONMENT (Subject Wise Compilation)

Harike is one of the largest man-made wetlands of northern India which shares its area with
the Tarntaran, Ferozpur and Kapurthala districts of Punjab. It came into existence in 1952
after the construction of barrage near the confluence of rivers Sutlej and Beas. Harike is a
significant abode for the birds migrating from across the international frontiers. The wetland
area is spread over about 41 km2 and supports more than 400 avian species. In addition to
haven for birds, Harike also harbours endangered aquatic mammalian as well as reptilian
fauna like Indus river dolphin, smooth-coated otter and seven species of rare freshwater
turtles. An area of about 86 km2 has been notified as wildlife sanctuary. Considered a
wetland of international importance especially as waterfowl refuge, this site was accorded
the wetland status in 1990 by the Ramsar Convention.

Q.131) Consider the statements about National Biodiversity Authority (NBA)

1. The NBA is a Statutory and Autonomous Body


2. It performs the function of regulatory as well as advisory body for GoI
3. The National Biodiversity Authority (NBA) was established in 2010 to implement
Indias Biological Diversity Act (2002).

Which of the given statements is/are correct?

a) 1 and 2
b) 2 and 3
c) 1 and 3
d) 1, 2 and 3

Q.131) Solution (a)

The National Biodiversity Authority (NBA) was established in 2003 to implement Indias
Biological Diversity Act (2002). The NBA is a Statutory, Autonomous Body and it performs
facilitative, regulatory and advisory function for the Government of India on issues of
conservation, sustainable use of biological resources and fair and equitable sharing of
benefits arising out of the use of biological resources.

The main objectives of NBA are:

To regulate access to biological resources of the country to conserve and sustainable use of
biological diversity.

To respect and protect the knowledge of local communities related to biodiversity

www.iasbaba.com Page 104


IASbabas 60 DAY PLAN- UPSC PRELIMS 2017
ENVIRONMENT (Subject Wise Compilation)

To secure sharing of benefits with the local people as conservers of biological resources,
holders of knowledge and information relating to the use of biological resources.

Conservation and development of area of importance from the view point of biological
diversity by declaring them as biological diversity heritage sites.

Protection and rehabilitation of threatened species, involvement of institutions of state


government in the broad scheme of implementation of the Biological Diversity Act through
constitution of committees.

Q.132) Consider the following

1. Durban Declaration and Program of Action


2. Cancum Agreement
3. Copenhagen Accord
4. Bali Action Plan

Which of the above includes commitments to reduce greenhouse gas emissions?

a) 1, 2 and 3
b) 2, 3 and 4
c) 2 and 3
d) All

Q.132) Solution (c)

Durban Declaration and Programme of Action- The DDPA reasserts the principles of
equality and non-discrimination as core human rights, thus transforming victims of
discrimination into rights-holders and States into duty bearers.

Bali Action Plan- The Bali Action Plan did not introduce binding commitments to reduce
greenhouse gas emissions but included the request for developed countries to contribute to
the mitigation of global warming in the context of sustainable development. In addition, the
Bali Action Plan envisaged enhanced actions on adaptation, technology development and on
the provision financial resources, as well as measures against deforestation.

Cancum Agreement

The agreements, reached on December 11 in Cancun, Mexico, at the 2010 United Nations
Climate Change Conference represent key steps forward in capturing plans to reduce

www.iasbaba.com Page 105


IASbabas 60 DAY PLAN- UPSC PRELIMS 2017
ENVIRONMENT (Subject Wise Compilation)

greenhouse gas emissions and to help developing nations protect themselves from climate
impacts and build their own sustainable futures.

The Copenhagen Agreement is a document that delegates at the 15th session of the
Conference of Parties (COP 15) to the United Nations Framework Convention on Climate
Change.

The Accord

Endorses the continuation of the Kyoto Protocol.


Underlines that climate change is one of the greatest challenges of our time and
emphasises a "strong political will to urgently combat climate change in accordance
with the principle of common but differentiated responsibilities and respective
capabilities"
To prevent dangerous anthropogenic interference with the climate system,
recognizes "the scientific view that the increase in global temperature should be
below 2 degrees Celsius", in a context of sustainable development, to combat
climate change.

Q.133) According to United Nations Convention on The Law of Seas (UNCLOS)

1. Territorial waters are up to 12 nautical miles.


2. Contiguous Zone is up to 22 nautical miles from the shore.
3. The coastal nation has sole exploitation rights over all natural resources in Exclusive
Economic Zones (EEZ).

Which of the above statements are incorrect?

a) None of the above


b) Only 2
c) 2 and 3
d) 1 and 2

Q.133) Solution (b)

The contiguous zone extends up to 24 nautical miles from the shore. In a contiguous zone
coastal nations can exercise their rights in four areas: customs, taxation, immigration and
pollution.

www.iasbaba.com Page 106


IASbabas 60 DAY PLAN- UPSC PRELIMS 2017
ENVIRONMENT (Subject Wise Compilation)

In Exclusive Economic Zone (EEZ), which extends up to 200 nautical mile, the coastal nations
have exclusive right to exploit the resources. International vessels are free to cross these
waters without any restriction.

Q.134) TRAFFIC, The Wildlife Trade Monitoring Network is a joint conservation


programme of

a) WWF and UNEP


b) WWF and UNDP
c) WWF and UN
d) WWF and IUCN

Q.134) Solution (d)

TRAFFIC, the wildlife trade monitoring network, is a joint program of WWF and IUCN the
International Union for Conservation of Nature. TRAFFIC works to ensure that trade in wild
plants and animals is not a threat to the conservation of nature. TRAFFIC has gained its
greatest reputation from supporting CITES, the Convention on International Trade in
Endangered Species. Since TRAFFIC was created in 1976 it has helped with the evolution of
this international wildlife trade treaty. TRAFFIC North America (NA) works through its offices
in the United States, Canada and Mexico, and with its programs in Central America and the
Caribbean.

TRAFFIC focuses on leveraging resources, expertise and awareness of the latest globally
urgent species trade issues such as tiger parts, elephant ivory and rhino horn. Large scale
commercial trade in commodities like timber and fisheries products are also addressed and
linked to work on developing rapid results and policy improvements.

Q.135) Consider the following regarding Joint Forest Management (JFM)

1. The policies and objectives of Joint Forest Movement are detailed in the Indian
Forest Act, 1927
2. At least 50% members of the JFM executive committee/management committee
should be women.
3. The presence of at least 50% women members should be a prerequisite for holding
the general body meeting.

Select the incorrect statement/s

www.iasbaba.com Page 107


IASbabas 60 DAY PLAN- UPSC PRELIMS 2017
ENVIRONMENT (Subject Wise Compilation)

a) 1 and 2
b) Only 2
c) 2 and 3
d) All of the above

Q.135) Solution (a)

Joint Forest Management often abbreviated as JFM is the official and popular term
in India for partnerships in forest movement involving both the state forest departments
and local communities. The policies and objectives of Joint Forest Movement are detailed in
the Indian comprehensive National Forest Policy of 1988 and the Joint Forest Movement
Guidelines of 1990 of the Government of India

The JFM programme in the country was reviewed by Government of India from time to time
in consultation with State Governments, NGOs and other stakeholders in view of several
emerging issues. In order to further strengthen the programme, the State Governments may
take action on the following suggested lines.

A) Legal backup to the JFM committees

i) At present, the JFM committees are being registered under different names in various
States as per the provisions contained in the resolutions. Except in a few States where the
committees are registered under the relevant acts in most of the states there is no legal
back up for these committees. It is therefore, necessary that all the State Governments
register the JFM or village committees under the Societies Registration Act, 1860 to provide
them with legal back up. This may be completed by 31st March, 2000. Completion of such
formation of existing JFM committees may please be reported to this Ministry.

ii) There are different nomenclatures for the JFM committees in different States. It would be
better if these committees are known uniformly as JFM committees (JFMC) in all the states.
Memorandum of Understanding, with clearly defined roles and responsibilities for different
work or areas should be separately assigned and signed between the State Governments
and the committees. All adults of the village should be eligible to become members of the
JFM Committees.

B) Participation of women in the JFM programme:

Considering the immense potential and genuine need for womens participation in JFM
programme, following guidelines are suggested for ensuring meaningful participation of
women in JFM.

www.iasbaba.com Page 108


IASbabas 60 DAY PLAN- UPSC PRELIMS 2017
ENVIRONMENT (Subject Wise Compilation)

i) At least 50% members of the JFM general body should be women. For the general body
meeting, the presence of at least 50% women members should be a prerequisite for holding
the general body meeting.

ii) At least 33% of the membership in the JFM Executive Committee/ Management
Committee should be filled from amongst the women members. The quorum for holding
meeting of such Executive/ Management Committee should be one-third of women
executive members or a minimum of one whichever is more. One of the posts of office
bearer i.e. President/ Vice-President/ Secretary should be filled by a women members of the
Committee.

Q.136) Pattiseema Lift Irrigation Project was in news recently. It connects

a) Mahanadi and Godavari


b) Krishna and Mahanadi
c) Krishna and Godavari
d) Periyar and Krishna

Q.136) Solution (c)

Pattiseema Lift Irrigation Scheme which has interlinked rivers Godavari and Krishna in West
Godavari district of Andhra Pradesh, has now got national level acknowledgment.

It is registered in Limca Book of Records for integrating Krishna and Godavari rivers within a
year time.

Important: It is South Indias first River Integration Project.

Q.137) Consider the following regarding Seaweeds

1. Seaweeds are rich in vitamins and minerals and are consumed as food in various
parts of the world
2. They are also used for the production of phytochemicals, viz., agar, carrageenan and
alginate.
3. Seaweed can be potentially used as organic fertilizers

Which of the given statements is/are correct?

a) 1 and 2

www.iasbaba.com Page 109


IASbabas 60 DAY PLAN- UPSC PRELIMS 2017
ENVIRONMENT (Subject Wise Compilation)

b) 2 and 3
c) Only 1
d) 1, 2 and 3

Q.137) Solution (d)

Seaweed cultivation, as a diversification activity in mariculture, has tremendous potential all


along the Indian coast. Seaweeds are rich in vitamins and minerals and are consumed as
food in various parts of the world and used for the production of phytochemicals, viz.,
agar, carrageenan and alginate, which are widely employed as gelling, stabilizing and
thickening agents in several industries of food, confectionery, pharmaceutical, dairy, textile,
paper, paint, etc.

In India, seaweeds are used as raw materials for the production of agar, alginate and liquid
seaweed fertilizer (LSF).

There are a few agar industries, algin industries and LSF industries situated at different
places in the maritime states of Tamil Nadu, Karnataka, Andhra Pradesh and Gujarat.

The red algae Gelidiella acerosa, Gracilaria edulis, G. crassa, G. foliifera and G. verrucosa are
used for agar manufacture and brown algae Sargassum spp., Turbinaria spp. and Cystoseira
trinodis for the production of alginates and liquid seaweed fertilizer.

The quantity of seaweeds exploited is inadequate to meet the raw material requirement of
Indian seaweed industries.

http://www.thehindu.com/todays-paper/tp-national/tp-andhrapradesh/seaweed-
cultivation-will-be-encouraged/article17791027.ece

Q.138) Consider the following about applications of Super Absorbent Polymers (SAP) in
Agriculture

1. SAPs are polluting and non-biodegradable and dont helps in reducing irrigation
frequency and water consumption.
2. They improves soil quality and resists drought stress
3. SAPs can reduce overuse of fertilizers and pesticides in fields.
4. SAPs act as soil matter flocculants.

Which of the given statements are correct?

a) 1, 3 and 4 Only

www.iasbaba.com Page 110


IASbabas 60 DAY PLAN- UPSC PRELIMS 2017
ENVIRONMENT (Subject Wise Compilation)

b) 2, 3 and 4 Only
c) 2 and 3 Only
d) 1, 2, 3 and 4

Q.138) Solution (b)

Super Absorbent Polymers, also known as SAP, hydrogel, absorbent polymers, absorbent
gels, super soakers, super slurpers, water gel, is a new type of macro molecular synthetic
water absorbing polymer material.

It has a water uptake potential as high as 100,000% of its own weight in a short period of
time by osmosis and form granules in soil to enhance soil properties.

SAPs are generally white sugar-like hygroscopic materials that swell in water to form a clear
gel made of separate individual particles and can retain moisture even under pressure
without risk of conflagration or rupturing/blasting.

Super Absorbent Polymers used in agriculture are mostly prepared from acrylic acids and a
cross-linking agent like potassium by solution or suspension polymerization. The polymer so
formed is called a polyacrylate whose swelling capacity and gel modulus depends greatly on
the quantity and type of cross-linker used.

Polyacrylates are non-toxic, non-irritating and non-corrosive in nature and tested to be


biodegradable with a degradation rate of 10%-15% per year. They demonstrate high water
absorbance potential and can freely release 95% of the same under suction pressure by
plant roots.

The three most common soil conditions that hinder plant growth and crop yield are low
water retention capability, high evapo-transpiration rate and soil moisture leaching. Apart
from these, factors like unforeseen drought conditions, degradation and salination, overuse
of synthetic fertilizers and pesticides and improper irrigation practices severely affect soil
and plants, often rendering permanent damage to soil biota as well.

Desirable characteristics for applications in agriculture

High absorption capacity in saline and hard water conditions


Optimized absorbency under load (AUL)
Lowest soluble content and residual monomer
Low price
High durability and stability in the swelling environment and during storage
Gradual biodegradability without formation of toxic species

www.iasbaba.com Page 111


IASbabas 60 DAY PLAN- UPSC PRELIMS 2017
ENVIRONMENT (Subject Wise Compilation)

pHneutrality after swelling in water


Photostability
Rewetting capability

Hydrogel agriculture technology, as it is popularly called, has the following advantages.

Improves soil quality, preserves water and resists drought stress


Increases seed sprouting and seedling development leading to better farm success
From the environmental aspects, it is non-polluting and biodegradable, helps in
reducing irrigation frequency and water consumption and creates a simple cyclic
process to provide water directly to roots and prevent soil compaction.
In agriculture and agroforestry, SAPs act as micro water reservoirs at plant roots.
They absorb natural and supplied water 400-500 times their own weight and release
it slowly on account of root capillary suction mechanism thus preventing water loss
in soil by leaching and evaporation.
SAPs form a consistent cyclic process of absorption and release of water; the water
so released can provide optimum moisture for quick germination and seedling
maturation. Thus it reduces seedling mortality by several folds in nurseries.
In cold regions, death during germination and maturation is common due to
moisture freezing in & around plant root tissue. Absorbed moisture in hydrogels
does not freeze and makes easy accessibility to plants. It also regulates seedling
growth temperature preventing death by freezing.
SAPs can help save water and labor by reducing irrigation frequency, help overcome
drought conditions and act as soil conditioners, prevent leaching in sandy soils,
runoffs in mountainous and sloping fields, improve virescence efficiency and restore
soil biota.
SAPs can reduce overuse of fertilizers and pesticides in fields. The chemicals so
absorbed with water are slowly released thus extending the operational life and
uptake efficacy by root systems.
SAPs act as soil matter flocculants. They closely bind loose soil thus forming loams
that can help better root latching. Simultaneously, the repeated absorb-release
mechanism prevents over compaction of soil minerals and provides space for
aeration and development of soil edaphon.
It has a wide area of application ranging from agriculture, forestry, industrial
planting, municipal gardening, drought management, water conservation, It helps
reduce soil erosion by surface run-offs, fertilizer and pesticide leaching to ground
water, reducing cost of water and irrigation and success rate at growth and high
yields of crops.

www.iasbaba.com Page 112


IASbabas 60 DAY PLAN- UPSC PRELIMS 2017
ENVIRONMENT (Subject Wise Compilation)

Q.139) Plant nutrition is a term that takes into account the interrelationships of mineral
elements in the soil or soilless solution as well as their role in plant growth. Consider the
following

1. Nickel
2. Molybdenum
3. Copper
4. Boron
5. Sulphur
6. Silicon
7. Vanadium

Which of the given above has a role of essential and beneficial mineral nutrients that are
crucial for plant nutrition and growth?

a) 2, 3, 5 and 6
b) 1, 2, 3, 4, 5 and 7
c) 2, 3, 4, 5, 6 and 7
d) 1, 2, 3, 4, 5, 6 and 7

Q.139) Solution (d)

The following is a brief guideline of the role of essential and beneficial mineral nutrients that
are crucial for growth. Eliminate any one of these elements, and plants will display
abnormalities of growth, deficiency symptoms, or may not reproduce normally.

Macronutrients

Nitrogen is a major component of proteins, hormones, chlorophyll, vitamins and enzymes


essential for plant life. Nitrogen metabolism is a major factor in stem and leaf growth
(vegetative growth). Too much can delay flowering and fruiting. Deficiencies can reduce
yields, cause yellowing of the leaves and stunt growth.

Phosphorus is necessary for seed germination, photosynthesis, protein formation and


almost all aspects of growth and metabolism in plants. It is essential for flower and fruit
formation. Low pH (<4) results in phosphate being chemically locked up in organic soils.
Deficiency symptoms are purple stems and leaves; maturity and growth are retarded. Yields
of fruit and flowers are poor. Premature drop of fruits and flowers may often occur.
Phosphorus must be applied close to the plant's roots in order for the plant to utilize it.
Large applications of phosphorus without adequate levels of zinc can cause a zinc
deficiency.

www.iasbaba.com Page 113


IASbabas 60 DAY PLAN- UPSC PRELIMS 2017
ENVIRONMENT (Subject Wise Compilation)

Potassium is necessary for formation of sugars, starches, carbohydrates, protein synthesis


and cell division in roots and other parts of the plant. It helps to adjust water balance,
improves stem rigidity and cold hardiness, enhances flavor and color on fruit and vegetable
crops, increases the oil content of fruits and is important for leafy crops. Deficiencies result
in low yields, mottled, spotted or curled leaves, scorched or burned look to leaves.

Sulfur is a structural component of amino acids, proteins, vitamins and enzymes and is
essential to produce chlorophyll. It imparts flavor to many vegetables. Deficiencies show as
light green leaves. Sulfur is readily lost by leaching from soils and should be applied with a
nutrient formula. Some water supplies may contain Sulfur.

Magnesium is a critical structural component of the chlorophyll molecule and is necessary


for functioning of plant enzymes to produce carbohydrates, sugars and fats. It is used for
fruit and nut formation and essential for germination of seeds. Deficient plants appear
chlorotic, show yellowing between veins of older leaves; leaves may droop. Magnesium is
leached by watering and must be supplied when feeding. It can be applied as a foliar spray
to correct deficiencies.

Calcium activates enzymes, is a structural component of cell walls, influences water


movement in cells and is necessary for cell growth and division. Some plants must have
calcium to take up nitrogen and other minerals. Calcium is easily leached. Calcium, once
deposited in plant tissue, is immobile (non-translocatable) so there must be a constant
supply for growth. Deficiency causes stunting of new growth in stems, flowers and roots.
Symptoms range from distorted new growth to black spots on leaves and fruit. Yellow leaf
margins may also appear.

Micronutrients

Iron is necessary for many enzyme functions and as a catalyst for the synthesis of
chlorophyll. It is essential for the young growing parts of plants. Deficiencies are pale leaf
color of young leaves followed by yellowing of leaves and large veins. Iron is lost by leaching
and is held in the lower portions of the soil structure. Under conditions of high pH (alkaline)
iron is rendered unavailable to plants. When soils are alkaline, iron may be abundant but
unavailable. Applications of an acid nutrient formula containing iron chelates, held in
soluble form, should correct the problem.

Manganese is involved in enzyme activity for photosynthesis, respiration, and nitrogen


metabolism. Deficiency in young leaves may show a network of green veins on a light green
background similar to an iron deficiency. In the advanced stages the light green parts
become white, and leaves are shed. Brownish, black, or grayish spots may appear next to
the veins. In neutral or alkaline soils plants often show deficiency symptoms. In highly acid
soils, manganese may be available to the extent that it results in toxicity.

www.iasbaba.com Page 114


IASbabas 60 DAY PLAN- UPSC PRELIMS 2017
ENVIRONMENT (Subject Wise Compilation)

Boron is necessary for cell wall formation, membrane integrity, calcium uptake and may aid
in the translocation of sugars. Boron affects at least 16 functions in plants. These functions
include flowering, pollen germination, fruiting, cell division, water relationships and the
movement of hormones. Boron must be available throughout the life of the plant. It is not
translocated and is easily leached from soils. Deficiencies kill terminal buds leaving a rosette
effect on the plant. Leaves are thick, curled and brittle. Fruits, tubers and roots are
discolored, cracked and flecked with brown spots.

Zinc is a component of enzymes or a functional cofactor of a large number of enzymes


including auxins (plant growth hormones). It is essential to carbohydrate metabolism,
protein synthesis and internodal elongation (stem growth). Deficient plants have mottled
leaves with irregular chlorotic areas. Zinc deficiency leads to iron deficiency causing similar
symptoms. Deficiency occurs on eroded soils and is least available at a pH range of 5.5 - 7.0.
Lowering the pH can render zinc more available to the point of toxicity.

Copper is concentrated in roots of plants and plays a part in nitrogen metabolism. It is a


component of several enzymes and may be part of the enzyme systems that use
carbohydrates and proteins. Deficiencies cause die back of the shoot tips, and terminal
leaves develop brown spots. Copper is bound tightly in organic matter and may be deficient
in highly organic soils. It is not readily lost from soil but may often be unavailable. Too much
copper can cause toxicity.

Molybdenum is a structural component of the enzyme that reduces nitrates to ammonia.


Without it, the synthesis of proteins is blocked and plant growth ceases. Root nodule
(nitrogen fixing) bacteria also require it. Seeds may not form completely, and nitrogen
deficiency may occur if plants are lacking molybdenum. Deficiency signs are pale green
leaves with rolled or cupped margins.

Chlorine is involved in osmosis (movement of water or solutes in cells), the ionic balance
necessary for plants to take up mineral elements and in photosynthesis. Deficiency
symptoms include wilting, stubby roots, chlorosis (yellowing) and bronzing. Odors in some
plants may be decreased. Chloride, the ionic form of chlorine used by plants, is usually
found in soluble forms and is lost by leaching. Some plants may show signs of toxicity if
levels are too high.

Nickel has just recently won the status as an essential trace element for plants according to
the Agricultural Research Service Plant, Soil and Nutrition Laboratory in Ithaca, NY. It is
required for the enzyme urease to break down urea to liberate the nitrogen into a usable
form for plants. Nickel is required for iron absorption. Seeds need nickel in order to
germinate. Plants grown without additional nickel will gradually reach a deficient level at
about the time they mature and begin reproductive growth. If nickel is deficient plants may
fail to produce viable seeds.

www.iasbaba.com Page 115


IASbabas 60 DAY PLAN- UPSC PRELIMS 2017
ENVIRONMENT (Subject Wise Compilation)

Sodium is involved in osmotic (water movement) and ionic balance in plants.

Cobalt is required for nitrogen fixation in legumes and in root nodules of nonlegumes. The
demand for cobalt is much higher for nitrogen fixation than for ammonium nutrition.
Deficient levels could result in nitrogen deficiency symptoms.

Silicon is found as a component of cell walls. Plants with supplies of soluble silicon produce
stronger, tougher cell walls making them a mechanical barrier to piercing and sucking
insects. This significantly enhances plant heat and drought tolerance. Foliar sprays of silicon
have also shown benefits reducing populations of aphids on field crops. Tests have also
found that silicon can be deposited by the plants at the site of infection by fungus to combat
the penetration of the cell walls by the attacking fungus. Improved leaf erectness, stem
strength and prevention or depression of iron and manganese toxicity have all been noted
as effects from silicon. Silicon has not been determined essential for all plants but may be
beneficial for many.

Vanadium may be required by some plants, but at very low concentrations. It may also be
substituting for molybdenum.

N - synthesis of proteins and part of chlorophyll molecule

P - energy transfer

K - carbohydrate metabolism, water relations

Mg - chlorophyll molecule

S - proteins

Ca - strength of cell wall

Micronutrients - enzyme activation

MOBILE IMMOBILE
N Ca
P S
K B
Mg Fe
Cu
Mn
Zn

www.iasbaba.com Page 116


IASbabas 60 DAY PLAN- UPSC PRELIMS 2017
ENVIRONMENT (Subject Wise Compilation)

Q.140) Consider the following

1. Apple
2. Banana
3. Buckwheat
4. Coffee
5. Mango
6. Carrot
7. Soybeans

Which of the given above are not self-pollinated?

a) 1, 2, 3, 4, 5 and 6
b) 2, 3, 6 and 7
c) 1, 3, 4, 5 and 6
d) 1, 3, 5, 6 and 7

Q.140) Solution (c)

The most essential staple food crops on the planet,


like corn, wheat, rice, soybeans and sorghum, need no insect help at all; they are wind-
pollinated or self-pollinating.

Other staple food crops, like bananas and plantains, are sterile and propagated from
cuttings, requiring no pollination of any form, ever.

Name Pollinators

Fagopyrum
Buckwheat Honey bees, Solitary bees
esculentum

Malus domestica, Honey bees, orchard mason


Apple
or Malus sylvestris bee, Bumblebees, Solitary bees , Hover flies

www.iasbaba.com Page 117


IASbabas 60 DAY PLAN- UPSC PRELIMS 2017
ENVIRONMENT (Subject Wise Compilation)

Coffea spp. Coffea


Coffea spp. Honey bees, Stingless bees, Solitary bees
arabica, Coffea canephora

Mango Mangifera indica Honey bees, Stingless bees, Flies, Ants, Wasps

Carrot Daucus carota Flies, Solitary bees, Honey bees

Hint- http://www.thehindu.com/life-and-style/food/new-buzz-in-town/article17846880.ece

http://www.thehindu.com/todays-paper/tp-national/tp-kerala/mango-farmers-rue-fall-in-
price/article18263086.ece

http://www.thehindu.com/opinion/columns/Decline-of-pollinators-threatens-food-
supply/article14181840.ece

Q.141) Consider the following regarding Parthenium

1. It is a highly prevalent weed endemic to India


2. It is highly poisonous and severely reduces the crop productivity besides loss to
biodiversity and environment.
3. The weed causes dermatitis and asthma to human beings.

Which of the given statements is/are incorrect?

a) 1 and 3
b) 2 Only
c) 1 Only
d) None

Q.141) Solution (c)

www.iasbaba.com Page 118


IASbabas 60 DAY PLAN- UPSC PRELIMS 2017
ENVIRONMENT (Subject Wise Compilation)

Parthenium hysterophorus L., commonly known as carrot weed, white top or congress grass
in India, is a herbaceous, erect and annual plant belonging to the family Asteracae
(compositae).

It is most popularly known as gajar ghas due to its appearance like carrot plant. The origin of
Parthenium is considered to be from Mexico, America, Trinidad to and Argentina. After
noticeable occurrence of Parthenium in Pune (Maharashtra) in 1956, it has spread like a wild
fire throughout India. At present it has invaded about 35 million hectares of land in India. It
is a nuisance on road sides and railway tracks, vacant lands, wastelands, industrial areas, on
the sides of open drainage system and irrigation canals besides invading agricultural crop.

In general, parthenium is a poisonous, pernicious, problematic, allergic and aggressive weed


posing a serious threat to human beings and livestock. In India and Australia, this weed has
been considered as one of the greatest source of dermatitis, asthma, nasal-dermal and
nasal-bronchial types of diseases. Besides ill effects, it also causes several other problems
like blockage of common pathways and reduces the aesthetic values of parks, gardens and
residential colonies.

Parthenium also infest every type of crop, orchards, plantations and forest. It severely
reduces the crop productivity besides loss to biodiversity and environment.

http://www.thehindu.com/news/cities/Madurai/Efforts-needed-to-eradicate-
parthenium/article14574270.ece

http://timesofindia.indiatimes.com/citizen-reporter/stories/they-came-across-an-
important-thing-is-this-morning-tee/crshow/57413527.cms

Q.142) Readiness for Investment in Sustainable Energy (RISE) is developed by

a) World Bank Group


b) United Nations Development Programme
c) United Nations Environment Programme
d) International Energy Agency

Q.142) Solution (a)

Readiness for Investment in Sustainable Energy (RISE), developed by the World Bank Group,
is a suite of indicators that assesses the legal and regulatory environment for investment in

www.iasbaba.com Page 119


IASbabas 60 DAY PLAN- UPSC PRELIMS 2017
ENVIRONMENT (Subject Wise Compilation)

sustainable energy. It establishes a framework for better depicting the national enabling
environment to attract investment into sustainable energy.

RISE is aimed at policymakers who focus on actions within their control. Creating this
environment is directed by policymakersthe primary constituency RISE aims to influence.
RISE supports the achievement of the objectives of the Sustainable Energy for All (SE4ALL)
initiative.

Q.143) The decisions on REDD+ enumerate some "eligible activities" that developing
countries may implement to reduce emissions and enhance removals of greenhouse
gases. Identify the activities

1. Reducing emissions from deforestation


2. Reducing emissions from land degradation
3. Conservation of forest carbon stocks
4. Sustainable management of non-renewable energy
5. Enhancement of forest carbon stocks

Select the correct code

a) 1, 2, 3 and 5
b) 1, 3 and 5
c) 1, 2, 3, 4 and 5
d) 2, 3, 4 and 5

Q.143) Solution (b)

What is REDD+?

Reducing emissions from deforestation and forest degradation in developing countries and
the role of conservation, sustainable management of forests and enhancement of forest
carbon stocks in developing countries (REDD+) encourages developing countries to
contribute to climate change mitigation in the forest sector through the following activities:

reducing emissions from deforestation


reducing emissions from forest degradation
conservation of forest carbon stocks
sustainable management of forests
enhancement of forest carbon stocks

www.iasbaba.com Page 120


IASbabas 60 DAY PLAN- UPSC PRELIMS 2017
ENVIRONMENT (Subject Wise Compilation)

Q.144) Consider the following statements regarding National Mission on Sustainable


Habitats:

1. It is one of the missions under National Action Plan on Climate Change.


2. It works for the development and protection of natural habitats, especially of those
species which are at the verge of extinction due to habitat loss.
3. The mission works on development of green corridors to connect protected areas of
India.

Which of the above statements are incorrect?

a) 1 and 2
b) 2 and 3
c) 3 only
d) None of the above

Q.144) Solution (b)

The National Mission on Sustainable Habitat is one of the missions under National Action
Plan on Climate Change and aims to make cities sustainable through improvements in
energy efficiency in buildings, management of solid waste & shift to public transport.

Q.145) Consider the following regarding Methanogens

1. They are found in ruminants of animals and human


2. They play significant role in anaerobic wastewater treatment
3. They produce oxygen, carbon dioxide, hydrogen sulphide and methane

Which of the given statements is/are correct?

a) 1, 2 and 3
b) 1 and 2
c) 2 and 3
d) None

Q.145) Solution (b)

www.iasbaba.com Page 121


IASbabas 60 DAY PLAN- UPSC PRELIMS 2017
ENVIRONMENT (Subject Wise Compilation)

They are common in wetlands, where they are responsible for marsh gas, and in the
digestive tracts of animals such as ruminants and humans, where they are responsible for
the methane content of belching in ruminants and flatulence in humans.

Moreover, the methanogenic archaea populations play an indispensable role in anaerobic


wastewater treatments. They are anaerobic organisms and cannot function under aerobic
conditions

Q.146) Match the following list of bioactive substances and their roles

Bioactive Substance Role

1. Statin i. Removal of oil stains


2. Cyclosporin A ii. Removal of clots from blood vessels
3. Streptokinase iii. Lowering of blood cholesterol
4. Lipase iv. Immuno-suppressive agent

Choose the correct match:

a) 1-iii, 2-iv, 3-ii, 4-i


b) 1-ii, 2-iv, 3-iii, 4-i
c) 1-i, 2-ii, 3-iii, 4-iv
d) 1-iv, 2-iii, 3-ii, 4-i

Q.146) Solution (a)

Q.147) World Sustainable Development Summit (WSDS) is a flagship initiative by

a) Food and Agriculture Organization (FAO)


b) United Nations Development Programme (UNDP)
c) The Energy and Resources Institutes (TERI)
d) UNESCO

Q.147) Solution (c)

The World Sustainable Development Summit (WSDS), TERI's flagship event, has been
conceptualized as a single platform to accelerate action towards sustainable development
and especially climate change. The WSDS series seeks to bring together the finest minds and

www.iasbaba.com Page 122


IASbabas 60 DAY PLAN- UPSC PRELIMS 2017
ENVIRONMENT (Subject Wise Compilation)

leading thinkers of the world to focus attention on the challenge of sustainable


development and has emerged as a landmark event addressing issues pertinent to the
future of humanity.

WSDS builds on the 15 years legacy of the Delhi Sustainable Development Summit (DSDS)
which was the leading forum for discussing sustainable development issues. The DSDS held
under the aegis of the Ministry of Environment, Forest and Climate Change with support
from the Ministry of External Affairs, Government of India was an epitome of Track 2
diplomacy .

With an aim of expanding the scope and reach of the Summit to the global community,
DSDS has now transitioned to WSDS.

WSDS 2016 was held in New Delhi from October 5-8, 2016 under the broad rubric of
'Beyond 2015: People, Planet & Progress', and it broadly focused on actions, on accelerated
implementation of SDGs and NDCs.

Q.148) United Nations has declared___________as the International Year of Sustainable


Tourism for Development

a) 2014
b) 2015
c) 2016
d) 2017

Q.148) Solution (d)

http://media.unwto.org/press-release/2017-01-03/2017-international-year-sustainable-
tourism-development

Q.149) National Mission for Sustaining the Himalayan Ecosystem comes under

a) Ministry of Earth and Science


b) Ministry of Environment and Forest
c) Ministry of Science and Technology
d) Ministry of Agriculture

Q.149) Solution (c)

www.iasbaba.com Page 123


IASbabas 60 DAY PLAN- UPSC PRELIMS 2017
ENVIRONMENT (Subject Wise Compilation)

http://knowledgeportal-nmshe.in/NAPCC.aspx

Q.150) The United Nations General Assembly formally adopted the "universal, integrated
and transformative" 2030 Agenda for Sustainable Development, a set of 17 Sustainable
Development Goals (SDGs). Identify the correct goals

1. Gender Inequality
2. Affordable and clean energy
3. Combat desertification
4. End of nuclear energy
5. Clean water and sanitation
6. Food security

Select the correct code

a) 1, 2, 3, 4, 5 and 6
b) 2, 3, 5 and 6
c) 1, 2, 3, 5 and 6
d) 1, 2, 3, 4 and 5

Q.150) Solution (b)

Goal 1: No Poverty
Goal 2: Zero Hunger
Goal 3: Good Health and Well-being
Goal 4: Quality Education
Goal 5: Gender Equality
Goal 6: Clean Water and Sanitation
Goal 7: Affordable and Clean Energy
Goal 8: Decent Work and Economic Growth
Goal 9: Industry, Innovation and Infrastructure
Goal 10: Reduced Inequalities
Goal 11: Sustainable Cities and Communities
Goal 12: Responsible Consumption and Production
Goal 13: Climate Action
Goal 14: Life Below Water
Goal 15: Life on Land
Goal 16: Peace, Justice and Strong Institutions

www.iasbaba.com Page 124


IASbabas 60 DAY PLAN- UPSC PRELIMS 2017
ENVIRONMENT (Subject Wise Compilation)

Goal 17: Partnerships for the Goals

www.iasbaba.com Page 125

You might also like